Return Home Members Area Experts Area The best AskMe alternative!Answerway.com - You Have Questions? We have Answers! Answerway Information Contact Us Online Help
 Sunday 28th March 2010 10:56:34 AM


 

Username:

Password:

or
Join Now!

 
These are answers that matrix has provided in Chemistry

Question/Answer
cheriskae asked on 01/31/08 - help

Using chemical test, how can you distinguish among 2-pentanone,3-pentanone and pentanal?
Can you also draw the structure of a compound, C5H8O that gives a positive iodoform test and does not decolorize permanganate?

matrix answered on 02/01/08:

Hello cheriskae

In the future please one problem per question.
..................................................................
(1)Using chemical test, how can you distinguish among 2-pentanone,3-pentanone and pentanal?
...

Only pentanal will give a positive Tollens test and only 2-pentanone will give a positive iodoform test.
...
Tollens reagent (silver mirror test) is used as a test for aldehydes, which are oxidized to carboxylic acids.
...
The iodoform test is a qualitative chemical test for the detection of ketones and aldehydes carrying an alpha methyl group. Alcohols with groups such CH3CH(OH)-R will also give positive test.
...
3-Pentanone will negative results in both of these tests.
....................................................................
(2) Can you also draw the structure of a compound, C5H8O that gives a positive iodoform test and does not decolorize permanganate?

...
Permanganate decoloration is used as a qualitative test for the presence of an alkene (Bayer test).
...
For C5H8O the only material that contains a methyl ketone and no unsaturation is cyclopropyl methyl ketone.
...
See webpage below for structure.
http://orgsyn.org/orgsyn/orgsyn/prepContent.asp?prep=cv4p0597

cheriskae rated this answer Excellent or Above Average Answer

Question/Answer
cheriskae asked on 01/31/08 - help

Using chemical test, how can you distinguish among 2-pentanone,3-pentanone and pentanal?
Can you also draw the structure of a compound, C5H8O that gives a positive iodoform test and does not decolorize permanganate?

matrix answered on 02/01/08:

Hello cheriskae

In the future please one problem per question.
..................................................................
(1)Using chemical test, how can you distinguish among 2-pentanone,3-pentanone and pentanal?
...

Only pentanal will give a positive Tollens test and only 2-pentanone will give a positive iodoform test.
...
Tollens reagent (silver mirror test) is used as a test for aldehydes, which are oxidized to carboxylic acids.
...
The iodoform test is a qualitative chemical test for the detection of ketones and aldehydes carrying an alpha methyl group. Alcohols with groups such CH3CH(OH)-R will also give positive test.
...
3-Pentanone will negative results in both of these tests.
....................................................................
(2) Can you also draw the structure of a compound, C5H8O that gives a positive iodoform test and does not decolorize permanganate?

...
Permanganate decoloration is used as a qualitative test for the presence of an alkene (Bayer test).
...
For C5H8O the only material that contains a methyl ketone and no unsaturation is cyclopropyl methyl ketone.
...
See webpage below for structure.
http://orgsyn.org/orgsyn/orgsyn/prepContent.asp?prep=cv4p0597

cheriskae rated this answer Excellent or Above Average Answer

Question/Answer
cheriskae asked on 01/25/08 - Please help

Instead of t-butyl alcohol, what other compounds can we use to form1,4-di-t-butyl-2,5-dimethoxybenzene?

matrix answered on 01/27/08:

In Friedel Crafts reaction there are 3 common kinds of reactants.
1) Alcohols
2) Bromo or chloro compounds. In your case you would use tert-butyl chloride or tert-butyl bromide.
3) Alkenes. In this case you would use isobutylene.

cheriskae rated this answer Excellent or Above Average Answer

Question/Answer
cheriskae asked on 01/24/08 - help me out please

Why is the 1,4 isomer, 1,4-di-t-butyl-2,5-dimethoxybenzene,the major product in the alkylation of dimethoxy benzene?Would you expect either of the following compounds to be formed as side products:1,3-di-t-butyl-2,5-dimethoxybenzene or 1,4-dimethoxy-2,3-di-t-butylbenzene? Why or why not?

matrix answered on 01/24/08:

The methoxy groups of 1,4-dimethoxybenzene are ortho,para directing. These direct the addition of the first tert-butyl group to an ortho position (the para positions are already occupied). The resulting compound now contains three ortho,para directing groups. The postion para to the first tert-butyl group is were the second tert-butyl group is directed.
.........
Electronic effects prevent the formation of 1,3-di-t-butyl-2,5-dimethoxybenzene.
........
1,4-Dimethoxy-2,3-di-t-butylbenzene is NOT formed because of steric effects. Two tert-butyl groups are too bulky to be on adjacent positions of the 1,4-dimethoxylbenzene.

cheriskae rated this answer Excellent or Above Average Answer

Question/Answer
cheriskae asked on 01/16/08 - Help please

What is the purpose of the phosphoric acid in "an alkene from alcohols" experiment? Why is phosphoric acid used rather than another strong acid such as hydrobromic acid?

matrix answered on 01/16/08:

Phosphoric acid is the catalyst in the reaction.
............
The dehydration of alcohols with acids is a general laboratory method for preparing alkenes. Strong mineral acids, such as sulfuric or phosphoric acids, usually are used as catalysts. Phosphoric acid rather than sulfuric acid is the preferred catalyst because
sulphuric acid produces messy results. Not only is it an acid, but it is also a strong oxidising agent. It oxidises some of the alcohol to carbon dioxide and at the same time is reduced itself to sulphur dioxide. Both of these gases have to be removed from the alkene usually by washing.

cheriskae rated this answer Excellent or Above Average Answer

Question/Answer
Jessica106 asked on 12/19/07 - Chemistry.. Calculating Mass

Someone please help me I am in Chemistry 20 and am having trouble understanding how to start solving this equation...


What mass of copper (II) hydroxide precipitate is produced by the reaction in solution of 2.67g of potassium hydroxide with excess aqueous copper (II) nitrate?

matrix answered on 12/19/07:

It helps to have a balanced equation for your question.
Cu(NO3)2 + 2 KOH ---> Cu(OH)2 + 2 KNO3
.......
Next determine the number of moles of your reactants.
2.67/56.1g/mole = 0.0476 moles of KOH
........
From your equation you can see that it takes two moles of to form one mole of Cu(OH)2.

2/1 = 0.0476/X
X = 0.0476/2 = 0.0238 moles of Cu(OH)2 will be formed.

0.0238 moles of Cu(OH)2 will have a mass of 2.32g
0.0238 moles x 97.6g/mole = 2.32g

Question/Answer
Ruth231 asked on 12/04/07 - Got some misunderstandings with this reaction:

6FeSO4+2NaNO3+4H2SO4=4H2O+2NO+Na2SO4+3Fe2(SO4)3 I suppose this reaction is right, but can't get the ionic reaction of this. I would appreciate any note about it.

matrix answered on 12/04/07:

6 FeSO4 + 2 NaNO3 + 4 H2SO4---->4 H2O + 2 NO + Na2SO4 + 3 Fe2(SO4)3
........
TOTAL IONIC EQUATION
6 Fe++ + 6 SO4-- + 2 Na+ + 2 NO3- + 8 H+ + 4 SO4-- ----> 4 H2O + 2 NO + 2 Na+ + SO4-- + 6 Fe+++ + 9 SO4--
.........
NET IONIC EQUATION
Subtract the ions that appear on both sides of the above equation.


6 Fe++ + 2 NO3- + 8 H+ ----> 4 H2O + 2 NO + 6 Fe+++

...........
The ionic equations are usually written with subscripts indicating if the species are in solution (aq), gas form (g) ,liquid (l) or solid( s).
6 Fe++(aq) + 2 NO3-(aq) + 8 H+(aq) ----> 4 H2O(l) + 2 NO(g) + 6 Fe+++(aq)

Ruth231 rated this answer Excellent or Above Average Answer

Question/Answer
lulu7 asked on 11/27/07 - Chem Lab--Help with Ideal Gas Law

My chem lab is dealing with the ideal gas law (and Dumas method). I have the mass, volume, temperature; however, I don't know how to find the pressure or number of moles. The volatile compound is unknown and I'm looking for the molar mass of the compound? Please HELP!!!

matrix answered on 11/27/07:


The pressure you use is the atmospheric pressure in the lab. Either read the barometer in the lab or ask your instructor.
..........
You calculate the number of moles from all the data you have.
PV = nRT
Solve for n.
n = PV/RT
...........
Use the correct value for R depending on the units of your data.
http://en.wikipedia.org/wiki/Gas_constant
............

Once you have the number of moles you will be able to determine the molar mass using the mass of your sample.
molar mass = mass/number of moles

lulu7 rated this answer Poor or Incomplete Answer

Question/Answer
Jaylene45 asked on 11/27/07 - Balancing equations

How do I balance this equation
KCl)3 = __KCl + O2

matrix answered on 11/27/07:




2 KClO3 ---> 2 KCl + 3 O2

Question/Answer
Jaylene45 asked on 11/27/07 - Noble Gas

Do noble gases make compounds?

matrix answered on 11/27/07:



Xenon reacts with fluorine to form a number of compounds.Examples are xenon difluoride (XeF2), xenon tetrafluoride (XeF4), xenon hexafluoride (XeF6), oxyfluorides(XeOF2, XeOF4, XeO2F2, XeO3F2, XeO2F4) and oxides (xenon trioxide (XeO3), xenon tetroxide (XeO4)).
.........
Krypton reacts with fluorine to form krypton fluoride KrF2.
..........
Argon fluoride (ArF2) has been reported but its discovery has not been confirmed.
...........
Radon reacts with fluorine to form radon fluoride (RnF2).

Question/Answer
xrayrod asked on 11/13/07 - Mass, Volume, and Density at STP

What mass of propane gass, C3H8, must be combusted to produce 120.L of carbon dioxide at 115*C and 0.854 atmosphere?

matrix answered on 11/14/07:

PV = nRT
Solve for n
n = PV/RT
n = (0.854)(120)/0.082057)(388K) = 3.22 moles
........
3.22 x 44.1 g/mole = 142 g
........
Gas constant
http://en.wikipedia.org/wiki/Gas_constant

Gas Laws
http://www.pinkmonkey.com/studyguides/subjects/chem/chap6/c0606801.asp



Question/Answer
misty712 asked on 11/14/07 - synthesis of K3[Fe(C2O4)3]3H2O

Hi,
In lab, Fe(NH4)2(SO4)2-6H2O was used to synthesize K3[Fe(C2O4)3]3H2O or K[Fe(C2O4)2]-2H2O. I need to show the reaction and balance it to show the two percent yields for each possible product. It says to use the 3 main reactants in the equation and then show how they form the product. I was assuming that the three main products were the iron ammounium sulfate hydrated salt + K2C2O4 + H2C2O4 because in the worksheet, the H2O2 and H20 are shown over the arrow (I am assuming as catalysts?) Then the product would be one of the two products + NH4 +SO4 + ??? maybe water? I am not sure how to write the reaction or balance it since the water is connected. Thanks!!!

matrix answered on 11/14/07:

Hello misty
I am not sure of your question.
From your question "I was assuming that the three main products were the iron ammonium sulfate hydrated salt + K2C2O4 + H2C2O4 ".

These are reactants (or starting materials),they are NOT products.

...........
I will give you the balanced equation and if your need more then give more details.
........
The iron is being oxidized by the hydrogen peroxide from Fe^2+ to Fe^3+.
.........
2 (NH4)2Fe(SO4)2 + 3 H2C2O4 + 4 H2O + H2O2 + 3 K2C2O4 -----> 2 K3[Fe(C2O4)3]·3H2O + 2 (NH4)2SO4 + 2 H2SO4

Question/Answer
slksrkhrr asked on 11/01/07 - Percent Yeild

If 1.65g of Cu(No3)2 is obtained from allowing 0.85g of Cu to react with excess HNO3. What is the percent Yield of the reaction. I have balnaced the equation as follows and tried to solve it but I got a weird answer. Please Help
CU(s)+4HNO3(aq)-->Cu(NO3)2(aq)+2H2)(l)+2NO2(g)

matrix answered on 11/01/07:




The correct equation is below. I believe you meant 2 H2O instead of 2 H2.
4 HNO3(aq) + Cu(s) ==> Cu(NO3)2(aq) + 2 NO2(g) + 2 H2O(l)
.................
0.850/63.5 = 0.0134 moles of Cu
You started with 0.0134 moles of Cu and this should form 0.0134 moles of Cu(NO3)2
................
0.0134 x 187.6 = 2.44 g of Cu(NO3)2 is theoretical yield.
.........
1.65/2.44 x 100 = 67.6% yield

slksrkhrr rated this answer Excellent or Above Average Answer

Question/Answer
cheriskae asked on 10/30/07 - Help

I don't know how to solve this : Help please
After a number of test runs, Helen can reliably get 55% yield in her synthesis of aspirin.How much aspirin would she make if she had as much acetic anhydride as she wanted, but only 4g of salicylic acid? What was her theoretical yield?

Salicylic acid MW 138.1
Acetic anhydride MW 102 density 1.08
Acetylsalicylic acid MW 180.15

matrix answered on 10/30/07:

4.00/138.1 = 0.0290 moles of salicylic acid
.........
This will give a theoretical yield of 0.0290 moles of aspirin.
.........
Since the actual yield is 55% then 0.0290 x 0.55 = 0.016 moles or 2.88g of aspirin

cheriskae rated this answer Excellent or Above Average Answer

Question/Answer
ReginaD asked on 10/29/07 - Stoichemistry

The reaction of salicylic acid with acetic anhydride yields aspirin and acetic acid. Waht is the theoretical yield of aspirin if 85 grams of salicylic acid is treated with excess acetic anhydride?

matrix answered on 10/29/07:

Determine the number of moles of salicylic acid.
85g/138 g/mole = 0.62 moles
.......
0.62 moles is the theoretical yield of aspirin.
0.62 moles x 180 g/mole = 112g

ReginaD rated this answer Excellent or Above Average Answer

Question/Answer
aliceclin asked on 10/28/07 - Douglasite

Hey matrix, I've got a questiong for you.

Douglasite is a mineral with the formula 2 KCl·FeCl2·2 H2O. Calculate the mass percent of douglasite in a 597.5 mg sample if it took 37.20 mL of a 0.1000 M AgNO3 solution to precipitate all of the Cl - as AgCl. Assume the douglasite is the only source of chloride ion.

I don't know how to balance it, if I need to? and how to find the mass percent.

matrix answered on 10/28/07:

Douglasite is 45.48 Cl-
35.45/311.88 x 100 = 45.48%Cl-
..........
37.20ml x 0.1000M = 3.720 mmoles of AgNO3 used and this is the number of mmoles of AgCl formed.
...........
3.720 x 35.45mg/mmole = 131.9mg of Cl- in your sample.
You know that 45.48 of Douglasite is Cl-.
45.48%X = 131.9
X = 131.9/0.4548 = 290.1 mg is weight of Douglasite in sample.
..........
290.1/597.4 x 100 = 48.55% Douglaste in sample.

aliceclin rated this answer Excellent or Above Average Answer

Question/Answer
aliceclin asked on 10/28/07 - metal identification

A 1.00 g sample of an alkaline earth metal chloride is treated with excess silver nitrate. All of the chloride is recovered as 1.38 g of silver chloride. Identify the metal.

I balanced the equation,
Cl + AgNO3 --> AgCl + NO3

what do I do now? how do I identify?
thanks

matrix answered on 10/28/07:

Remember to put charges on ions when you balance equations
Cl- + Ag+ ----> AgCl
............

You need the percent of Cl- in AgCl
35.45/ 143.3 x 100 = 24.74%
In your sample of 1.38g you have 0.341g of Cl-

.......
Since you started with 1.00g of unknown chloride and you know it contains 0.341g of Cl-
the starting material is an alkaline earth chloride that contains 34.1% Cl- and 65.9% X.
You need to decide which of the alkaline earth metal chlorides(BeCl2,CaCl2,MgCl2,SrCl2,BaCl2,RaCl2) has these percentages.

The alkaline earth metal chloride closest to these percentages is BaCl2.

..........
The website below is useful for calculating percentages.
http://www.ch.cam.ac.uk/magnus/MolWeight.html

aliceclin rated this answer Excellent or Above Average Answer

Question/Answer
aliceclin asked on 10/28/07 - Oxidation and Reduction

If 2.02 g of copper(II) chloride dihydrate is reduced by Mg(s), what is the minimum mass of Mg(s) required for this reduction?

all I could think to do is:
CuCl2 + Mg(2+) __> MgCl2 + Cu(2+)

thanks

matrix answered on 10/28/07:

Why did you use Mg(2+) in your equation????
Mg metal is zero.
The correct solution is below.




CuCl2 + Mg ---> Cu + MgCl2

The molar weight of CuCl2•2H2O is 170.48

Calculate the number of moles of CuCl2•2H2O
2.02/170.48 = 0.0118 moles

It requires one mole of Mg for each mole of CuCl2•2H2O.
You need 0.0118 moles of Mg
0.0118 x 24.31 = 0.287 g of Mg

aliceclin rated this answer Excellent or Above Average Answer

Question/Answer
chem142 asked on 10/28/07 - Stoichiometry

If 2.02 g of copper(II) chloride dihydrate is reduced by Mg(s), what is the minimum mass of Mg(s) required for this reduction?

matrix answered on 10/28/07:

CuCl2 + Mg ---> Cu + MgCl2

The molar weight of CuCl2•2H2O is 170.48

Calculate the number of moles of CuCl2•2H2O
2.02/170.48 = 0.0118 moles

It requires one mole of Mg for each mole of CuCl2•2H2O.
You need 0.0118 moles of Mg
0.0118 x 24.31 = 0.287 g of Mg

chem142 rated this answer Excellent or Above Average Answer

Question/Answer
bkmorgan asked on 10/27/07 - Standard Temperature and Pressure

How many moles of propane (C3H8) would a 2.50 L tank contain at STP?

matrix answered on 10/27/07:

One mole of any gas has a volume of 22.4 L at STP.

2.50/22.4 = 0.112 moles

Question/Answer
marietoma asked on 10/23/07 - help with chem please

How many grams of Kl are in 25.0 mL of a 3.0 % (m/v) Kl solution?

matrix answered on 10/23/07:

A solution that is 3.0% (m/v) contains 3.0 g of solute in 100 ml of solution.

3.0 x (25/100) = 0.75 g of KI

Question/Answer
cheriskae asked on 10/21/07 - help

If you have ligroine and cyclohexane ,how do you know which is saturated or unsaturated?

matrix answered on 10/22/07:

Hello
This is a strange question because ligroin(e) is a material of variable composition.It contains many different saturated hydrocarbons. It may contain some cyclohexane.


Ligroine and cyclohexane are both saturated.


Below is a textbook definition of ligroine:
"Ligroin is a refined saturated hydrocarbon petroleum fraction similar to petroleum ether used mainly as a laboratory solvent. It predominantly consists of C7 through C11 in the form of about 55% paraffins, 30% monocycloparaffins, 2% dicycloparaffins and 12% alkylbenzenes. It is nonpolar. Generally laboratory grade ligroin boils at 60 to 90 °C, but the following fractions of petroleum ether are commonly available: 30 to 40 °C, 40 to 60 °C, 60 to 80 °C, 80 to 100 °C and sometimes 100 to 120 °C. The 60 to 80 °C fraction is often used as a replacement for hexane."

cheriskae rated this answer Excellent or Above Average Answer

Question/Answer
cheriskae asked on 10/20/07 - help please

Cyclohexene reacts with a high concentration of bromine to give what coumpound? What is its stereochemistry?
I don't know how to draw a stucture of the compound formed when cyclohexene dissolves in a concentrated sulfuric acid.


matrix answered on 10/20/07:

Cyclohexene reacts with bromine to give trans-1,2-dibromocyclohexane.
See website below for a drawing of the structure.
http://www.uni-regensburg.de/Fakultaeten/nat_Fak_IV/Organische_Chemie/Didaktik/Keusch/D-Addit_ar-e.htm

Cyclohexene reacts with sulfuric acid to form cyclohexyl hydrogensulfate.This is soluble in sulfuric acid.

C6H5-HSO4

cheriskae rated this answer Excellent or Above Average Answer

Question/Answer
cheriskae asked on 10/20/07 - Question

If I combine Pinene with permanganate, is it saturated or unsaturated? how do I know?
Same question by combining paraffin + permanganate, paraffin +sulfuric acid

matrix answered on 10/20/07:

The pinene makes the purple permanganate turn yellow-brown. This is a positive test for unsaturation.
(The -ene ending of pinene gives you a hint that it is unsaturated)

Paraffin is a saturated material and will give negative permanganate and sulfuric acid tests.

cheriskae rated this answer Excellent or Above Average Answer

Question/Answer
cheriskae asked on 10/12/07 - Help

After doing distillation, if you consider dividing up the distillate collected into thirds: the first third, the middle, and the last third.What can you conclude about the purity of the first and last from the simple distillation, and from the fractional distillation, in other words will they have the same purity?

matrix answered on 10/14/07:

Hello cheriskae

This is a difficult question to answer because of the variables involved. I will give a simple answer.Let me know if you need a more detailed answer.


In a simple distillation where only one fraction is collected then if you divide the distillate into three parts, each of these parts will have the same purity.

In a fractional distillation where you collect fractions based on increasing boiling point then if you collect three fractions, the first will consist of mainly the material with the lower boiling point. The second fraction will be a mixture of the two materials and the third will be mainly the material having the higher boiling point.

cheriskae rated this answer Excellent or Above Average Answer

Question/Answer
RWICKENHEISER asked on 10/07/07 - SHOW HOW TO GET THIS BALANCED EQUATION

I HAVE THE ORIGINAL QUESTION AND THE ANSWER. HOW DO I GET THE BALANCED EQUATION. I NEED TO KNOW THE STEPS.

Al+ MnO4=MnO2+Al(OH)4

BALANCED EQUATION

Al+MnO4+2H2O=MnO2+Al(OH)4

matrix answered on 10/07/07:

First neither of your equations are correct.
You must indicate the charges.
It is MnO4- and Al(OH04- There should be a minus charge on each of these two materials.

The correctly balanced equation is
Al + MnO4- + 2 H2O ----> MnO2 + Al(OH)4-

The steps used to balance reactions such as this are shown on the following website.

http://www.helium.com/tm/596616/faced-redox-reaction-sadly

Please read this to understand the steps.
Ask me if you do not understand.

Question/Answer
lulu7 asked on 09/30/07 - Limiting Reactants

1.85 mol NaOH and 1.00 mol CO2 react as follows:
2 NaOH(s) + CO2(g) = Na2CO3(s) + H2O (l)
a. What is the limiting reactant?

b. How many moles of Na2CO3 can be produced?

c. How many moles of excess reactant remain after the completion of the reaction?

Please answer what you can and show your work -- I'm completely lost on this!

matrix answered on 10/01/07:



2 NaOH(s) + CO2(g) ----> Na2CO3(s) + H2O (l)

(a) From your equation you need 2 moles of NaOH for each mole of CO2. Since you only have 1.85 moles of NaOH it is the limiting reagent.

(b) The CO2 is the excess reagent.
2/1 = 1.85/X
X = (1.85 x 1)/2 = 0.925 moles of CO2 is used in the reaction. This will make 0.925 moles of Na2CO3.

(c) CO2: 1.00 - 0.925 = 0.075 moles of CO2 remain unreacted.

lulu7 rated this answer Excellent or Above Average Answer

Question/Answer
cheriskae asked on 09/30/07 - Help please

What will be the appearance of a TLC plate if a solvent of too low polarity is used for the development? too high polarity?

Why does the lid of the jar need to be on while developing your TLC plate?

What problem will ensue if the level of the developing liquid is higher than the applied spot in a TLC analysis?

matrix answered on 09/30/07:

(1)What will be the appearance of a TLC plate if a solvent of too low polarity is used for the development? too high polarity?

TOO LOW POLARITY. The spots will stay on or too near the origin.
TOO HIGH POLARITY. The spots will be at the top of the plate (solvent front).
In thin layer chromatography you want the spots to have a Rf of between 0.2 and 0.8.

(2)Why does the lid of the jar need to be on while developing your TLC plate?

The reason for covering the jar is to make sure that the atmosphere in the beaker is saturated with solvent vapor.
To help this, the jar is often lined with some filter paper soaked in solvent.Saturating the atmosphere in the jar
with vapor stops the solvent from evaporating as it rises up the plate. The solvent needs to go up the plate to NEAR the top.

(3)What problem will ensue if the level of the developing liquid is higher than the applied spot in a TLC analysis?

The spot will NOT move up the plate but will spread out as blob and contaiminate the solvent in the jar.

cheriskae rated this answer Excellent or Above Average Answer

Question/Answer
lulu7 asked on 09/30/07 - Chemical Reaction

Is Fe2O3+3 CO=2 Fe+3 CO2 a precipitation reaction, an acid-base reaction, or an oxidation-reduction reaction and how do you know?

matrix answered on 09/30/07:

In oxidation-reduction reactions something is oxidized and something is reduced. This is not true for the other kinds of reactions.



Fe2O3 + CO ----> Fe + CO2
In this reaction the iron is being reduced (going from +3 to zero) and the carbon is being oxidized (going from +2 to +4).

lulu7 rated this answer Excellent or Above Average Answer

Question/Answer
Wendy0920 asked on 09/23/07 - Chemistry equation

Use the following balanced equation:
2 C6H14 + 19 O2 -- 12 CO2 + 14 H2O

How many grams of C6H14 are required to produce 1.5 moles of carbon dioxide?

matrix answered on 09/25/07:

2 C6H14 + 19 O2 --> 12 CO2 + 14 H2O
2 moles of hexane gives 12 moles of carbon dioxide then X moles will give 1.5 moles.
2/12 = X/1.5
X = 3/12 = 0.25 moles
0.25 moles of hexane is 21.6
0.25 x 86.2 = 21.6 g
This should be rounded to 22 grams because you have only two significant figures.

Question/Answer
twoego asked on 09/25/07 - Chemistry

What is the mass in grams if 30.0 ml of a liquid if its density is 0.60g/ml?

matrix answered on 09/25/07:

Density = mass/volume
Solve for mass
Mass = density X volume

30.0 ml X 0.60g/ml = 18 grams

Question/Answer
cheriskae asked on 09/23/07 - Help please- Organic chemistry

Why do we have to ice a tea solution before moving on to the extraction with dichloromethane?
And what is the purpose of adding calcium chloride to dichloromethane after extracting?

matrix answered on 09/23/07:

Dichloromethane has a boiling point of 40ºC.You need to cool the tea solution before mixing with the dichloromethane so it will boil out or release a large amount of dichloromethane vapor into your face and the lab.

The calcium chloride is used as a drying agent. It removes the small amount of water dissolved in the dichloromethane.

cheriskae rated this answer Excellent or Above Average Answer

Question/Answer
KatiBabi asked on 09/23/07 - Moles of O2 that are needed to burn 1.5 mol of C8H18

2 C8H18(l) + 25 O2(g) 16 CO2(g) + 18 H2O(l)

(a) How many moles of O2 are needed to burn 1.50 mol of C8H18?

matrix answered on 09/23/07:

Hello KalBabi
Once you have a correctly balanced equation questions such as this become simple math questions.
It takes 25 moles of O2 for 2 moles of octane then it will take X moles for 1.50 moles of octane.

25/2 = X/1.5
Solve for X
X = (25x1.5)/2 = 18.75
This should be rounded to 18.8 moles of O2 because of the number of significant figures you have with 1.50 (3 significant figures).

2 C8H18(l) + 25 O2(g)---> 16 CO2(g) + 18 H2O(l)

Question/Answer
zardaz asked on 09/20/07 - Chem Help
Chem help

how many C atoms are in 5.00 grams of benzene C6H6?

matrix answered on 09/20/07:

5.00g /78.1g/mole = 0.0640 moles of benzene.
Since benzene has 6 carbon atoms per mole then you have 6 x 0.0640 = 0.384 moles of carbon atoms in your sample.

1 mole of carbon contains 6.022 x 10^23 atoms.

In 5.00 g of benzene there are
0.384 x 6.022 x 10^23 = 2.31 x 10^23 carbon atoms

Question/Answer
bmise24 asked on 09/20/07 - Calculation Help, Not sure where to start?

Barium oxide reacts with hydrochloric acid producing barium chloride and water. Calculate how much water (g) is produced from 35g of barium oxide and 25g of hydrochloric acid.

Balanced equation = BaO + 2HCl - BaCl2 + H2O

matrix answered on 09/20/07:

BaO + 2HCl----> BaCl2 + H2O
From your equation one mole of BaO gives one mole of H2O.

BaO: 35g / 153g/mole = 0.23 moles
This will make 0.23 moles of H2O

0.23moles X 18g/mole = 4.1 g of H2O


HCl: 25g / 36.5 = 0.68 moles
From equation you need two moles of HCl to react with each mole of BaO.
For 0.23 moles of BaO you would need 0.46 moles of HCl. Since you have 0.68 moles of HCl you have more HCl than you need. The limiting reagent is the BaO.




Question/Answer
Zenz04 asked on 09/13/07 - Balanced equation help?

Can you help me with the balanced equation for dissolving an Alka Seltzer tablet in water? I think it's:

C15NaH19O11CO3?(aspirin,citric acid,sodium bicarbonate, is an Alka Seltzer tablet I think)+H2O = C02+Na3C6H5O7

matrix answered on 09/13/07:

The aspirin (acetylsalicylic acid) is not an active ingredient in the effervescent action of Alka-Seltzer, but the baking soda (sodium bicarbonate) and citric acid are active.

C6H8O7 + 3 NaHCO3--→ 3 H2O + 3 CO2 + Na3C6H5O7

Question/Answer
Chance asked on 09/10/07 - Chemistry

what is the formula for compound containing lithium and floride, caloium and flurino, maggnesium and oxygen?

matrix answered on 09/10/07:

LiF

CaF2

MgO

Question/Answer
Zenz04 asked on 09/08/07 - I'm stuck and need help bad.

A certain mass of sodium hydroxide is required to produce 3 moles of water in the reaction 2NaOH + CO2 = Na2CO3 + H2O. Which of the following relationships determines the mass of NaOH required?

a. 3 mol H2O x (40g NaOH/2 mol NaOH) x (1 mol H20/18gH2O)

b. 3 mol H2O x (1 mol H20/2 mol NaOH) x (1 mol NaOH/40g NaOH)

c. 3 mol H2O x (2 mol NaOH/1 mol H2O) x (40g NaOH/1 mol NaOH)

d. 2 mol NaOH x (3 mol H2O/2 mol NaOH) x (18g H2)/1 mol H20)

I think it's D. But I'm not to sure.

matrix answered on 09/09/07:

Answer (c) is correct.

From equation:
For 1 mole of H2O you need 2 moles of NaOH.

If you want 3 moles of H2O then you need 3 x 2 moles of NaOH ---> 6 moles of NaOH.

1 mole of NaOH is 40g
6 moles x 40g/mole = 240 g NaOH

Zenz04 rated this answer Excellent or Above Average Answer

Question/Answer
cheriskae asked on 09/09/07 - Help Please


Sally is looking for a recrystallization solvent for an organic compound she just synthesized. What solubility properties should she be looking for an ideal recrystallization solvent?

matrix answered on 09/09/07:

A good recrystallization solvent should:

(1) dissolve a moderate quantity of the substance to be purified at an elevated temperature, but only a small quantity at lower temperatures
(2) not react with the material to be purified
(3) either dissolve impurities readily at a low temperature or not dissolve them at all
(d) be easily removable from the purified product.This last requirement usually means that the solvent must have a relatively low boiling point and evaporate readily.

Read for more details.
http://designer-drug.com/pte/12.162.180.114/dcd/chemistry/equipment/recrystallization.html

cheriskae rated this answer Above Average Answer

Question/Answer
frejalaub asked on 09/02/07 - net ionic help


aqueous solutions of K2C2O4 and CuSO4 are mixed to form a precipitate. I thought maybe CuCO3 + CO2 were the products but I can't get it balance. Please help.

matrix answered on 09/02/07:

The solid is cupric potassium oxalate.

2 K2C2O4 + CuSO4 ----> K2Cu(C2O4)2 + K2SO4

frejalaub rated this answer Excellent or Above Average Answer

Question/Answer
horban asked on 08/30/07 - I need HELP! please show me formula and how to solve this.

a 1.543g sample containing sulfate ion was treated with barium cholride reagent, and 0.2243 grams of barium sulfate was isolated. Calculate the percentage of sulfate ion in the sample.

please show me how to solve this

matrix answered on 08/30/07:

M.Wt of BaSO4 is 233.395
At.Wt. of Ba is 137.904
137.904/233.395 x 100 = 58.84%
Ba is 58.84%
SO4 is 41.16%

0.2243g x 41.16% = 0.09232g is amount of SO4-- in sample.
0.09232/1.543 x 100 = 5.98% This is % of SO4-- in sample.

Question/Answer
horban asked on 08/30/07 - oxidation state

What is the oxidation state of aluminum in KAl(SO4)2 (12H2O)?

matrix answered on 08/30/07:

KAl(SO4)2*(12H2O)
The oxidation number of Aluminum (Al) is +3.

horban rated this answer Excellent or Above Average Answer

Question/Answer
[quinn] asked on 08/15/07 - balanced chemical equation

could you please help me with the balanced chemical equation for the complete combustion of 2,2-dimethylhexane?

matrix answered on 08/15/07:

Hello Quinn

All the carbon is converted to CO2 and all the hydrogen is converted to H2O.

2 C8H18 + 25 O2 ---> 16 CO2 + 18 H2O

[quinn] rated this answer Excellent or Above Average Answer

Question/Answer
lapaix4mum asked on 07/05/07 - acids bases and salts

why is (Na2O)Na crayying 2 rather than O

matrix answered on 07/05/07:

Sodium in its compounds has a valence of +1
Oxygen has a valence of -2.

It takes two sodium ions [2 x (+1)] to balance the -2 charge of of the oxygen.
This is why the formula is Na2O.

Question/Answer
balla4791 asked on 06/03/07 - how many grams of KNO3 should be used to prepare a 2.00L of a .500M solution


Questions

how many grams of KNO3 should be used to prepare a 2.00L of a .500M solution?


matrix answered on 06/05/07:

The molecular weight of KNO3 is 101.10

One liter of 0.500M will contain 101.10/2 = 50.6 grams of KNO3.
Two liters will contain 101 grams.

Question/Answer
ritu90 asked on 06/04/07 - isomers

hiii matrix....
would u help me out with 2 questions...??..thanks
Draw and name all the isomers of C8H18 which hav single side chain.
Draw and name all the isomers of C8H18 which have two side chains.

thank you
ritu

matrix answered on 06/05/07:

I cannot draw structures with my keyboard but I will list the compounds. The names tell you the structures.For example, for 2-methylheptane first draw 7 carbons in a line (heptane) then put a carbon on the second carbon in the line. Finally fill in with hydrogens.

Single Side Chain
2-methylheptane
3-methylheptane
4-methylheptane
3-ethylhexane



Double Side Chain
2,2-dimethylhexane
2,3-dimethylhexane
2,4-dimethylhexane
2,5-dimethylhexane
3,3-dimethylhexane
3,4-dimethylhexane
3-ethyl-2-methylpentane
3-ethyl-3-methylpentane

ritu90 rated this answer Excellent or Above Average Answer

Question/Answer
dredling asked on 05/24/07 - Double displacement question?

I have a double displacement of NaOH and NiCl2 and I need to write a balaced equation, but I'm unsure how to get it started as far as what elements will be displaced.

matrix answered on 05/24/07:

2 NaOH + NiCl2 ---> 2 NaCl + Ni(OH)2

http://chemistry.about.com/cs/generalchemistry/a/aa072103a.htm

http://dl.clackamas.edu/ch104-04/double.htm

Question/Answer
lizarl asked on 05/23/07 - Zinc chloride

Zn + 2HCl → ZnCl2 + H2

If 0.600 gram of zinc is used, what is the amount of zinc chloride that is produced in the reaction?

matrix answered on 05/23/07:

0.600/65.4 = 0.00917 moles of zinc

This will make 0.00917 moles of ZnCl2

0.00917 x 136.3 = 1.25 grams of ZnCl2

Question/Answer
lfcsg asked on 05/22/07 - chem help again


A zinc powder is added to a solution of copper(II) sulfate. Which of the following best represents the balanced, net ionic equation?

a. Cu2+(aq) + Zn(s) Zn2+(aq) + Cu(s)
b. CuSO4(aq) + Zn(s) ZnSO4(aq) + Cu(s)
c. H+(aq) + OH-(aq) H2O(aq)
d. Cu2+(aq) + Zn2+(aq) Zn2+(aq) + Cu2+(aq)
e. CuSO4(aq) + Zn(aq) ZnSO4(aq) + Cu(aq)

matrix answered on 05/22/07:

A zinc powder is added to a solution of copper(II) sulfate. Which of the following best represents the balanced, net ionic equation?

a. Cu2+(aq) + Zn(s)----> Zn2+(aq) + Cu(s)

Question/Answer
BxDiamond asked on 05/10/07 - Balancing oxidation reduction reactions by the ion electron method

Identify the oxidation and reduction half reactions, the elements oxidized and reduced and oxidizing and reducing agents fot the reaction.

In Acid

Cu+NO3-> Cu+2 +NO2

matrix answered on 05/11/07:

Cu(s) + HNO3(aq) --> Cu(NO3)2(aq) + NO2(g)

Step 1: Net ionic equation
Cu(s) + H+(aq) + NO3-(aq) ---> Cu2+(aq) + NO3-(aq) + NO2(g)

Step 2 & 3: Half-reactions
Cu(s) ---> Cu2+(aq) (oxidation,Cu is oxidized)

NO3-(aq) ---> NO3-(aq) + NO2(g) (reduction,nitrogen is reduced)

Copper is the reducing agent. It is oxidized in the reaction.
Nitrogen is the oxidizing agent. It is reduced in the reaction.

Step 4: Mass Balancing
Cu(s) ---> Cu2+(aq) (oxidation)

2NO3-(aq) + 2H+(aq) ---> NO3-(aq) + NO2(g) + H2O(l) (reduction)

Step 5: Charge Balancing
Cu(s) ---> Cu2+(aq) + 2e- (zero charge on both sides)

2NO3-(aq) + 2H+(aq) + e- ---> NO3-(aq) + NO2(g) + H2O(l) (-1 charge on both sides)

Step 6: Multiply to equalize electrons (multiply the reduction equation by 2)
Cu(s) ---> Cu2+(aq) + 2e-

4NO3-(aq) + 4H+(aq) + 2e- ---> 2NO3-(aq) + 2NO2(g) + 2H2O(l)

Step 7: Add the reactions (electrons cancel)
4NO3-(aq) + 4H+(aq) + Cu(s) ---> Cu2+(aq) + 2NO3-(aq) + 2NO2(g) + 2H2O(l)

Step 8: Check atoms and charge are balanced

Step 9: Rewrite equation in molecular form
4HNO3(aq) + Cu(s) ---> Cu(NO3)2(aq) + 2NO2(g) + 2H2O(l)

Some websites that may help.
http://www.chemtutor.com/redox.htm

http://preparatorychemistry.com/Bishop_Balancing_Redox.htm

http://www2.wwnorton.com/college/chemistry/gilbert/concepts/chapter5/ch5_6.htm


Question/Answer
lfcsg asked on 05/05/07 - chem

anyone know the concentration of NH4+ in a 2.528 M (NH4)2SO4 solution

matrix answered on 05/05/07:

2 x 2.528 = 5.056 M in NH4+.

lfcsg rated this answer Excellent or Above Average Answer

Question/Answer
cod143 asked on 04/25/07 - Net Ionic Equations

Write molecular, ionic, and net ionic equations for the neutralization of magnesium carbonate, MgCO3 (Tums and Di-Gel), with HCl.

thanks!

matrix answered on 04/25/07:

MOLECULAR
MgCO3 + 2 HCl ----> MgCl2 + CO2 + H2O

IONIC
MgCO3(s) + 2 H+(aq) + 2 Cl-(aq) -----> Mg++(aq) + 2 Cl-(aq) + H2O + CO2(g)

NET IONIC
MgCO3(s) + 2 H+(aq) ---> Mg++(aq) + H2O + CO2(g)

Question/Answer
mcmufftcp asked on 04/11/07 - Preparation of Aspirin

Hi-
I'm going to be doing a prep of aspirin lab. To test the purity of the aspirin, we first have to test phenol, 2-propanol, benzoic acid and salicylic acid with [Fe(III)(NO3)3] to see which will bind with Fe3+ by turning the soln purple. I know that Fe3+ will bind with the -OH of phenol and not benzoic acid because it is a carboxylic acid. Will Fe3+ bind with the -OH in 2-propanol and salicylic acid??
Next, we test the purity of our aspirin vs a commercial grade by seeing if the ASA will bind with Fe3+. I think the commercial grade will for sure NOT bind with the Fe3+. If the experimentally prep ASA does, does that mean it has salicylic acid impurities in it???
Thank you for any insight you may be able to provide.

matrix answered on 04/11/07:

Phenol and salicylic acid will give a positive test. Salicylic acid is a phenol.

Benzoic acid and 2-propanol will NOT give a positive test.

The Fe+++ ion test is for phenols NOT for alcohols or carboxlic acids.

Pure aspirin will NOT give positive test.

Impure aspirin will give positive test if it contains some unreacted salicylic acid.

Old commercial aspirin that has been stored in moist air for long periods will probably give a positive test because some of the acetylsalicylic acid is hydrolyzed to salicylic acid and acetic acid.

mcmufftcp rated this answer Excellent or Above Average Answer

Question/Answer
jc6391 asked on 04/11/07 - please help

calculate the volume in liters.
2.00 mol of H2 at 300 K and 1.25 atm.

matrix answered on 04/11/07:

PV = nRT

Solve for V using following values.
V = nRT/P = 39.4 liters


n = 2.00 moles
R = 0.082057 [L · atm / K · mol]
T = 300K
P = 1.25 atm

R is universal gas constant.
See chart in following website.Use value that has same units as your problem.

http://en.wikipedia.org/wiki/Gas_constant

Question/Answer
erikaj2 asked on 04/10/07 - double replacement rxn

How do you predict this double replacement reaction: barium chloride trihydrate + silver nitrate?

matrix answered on 04/10/07:

BaCl2 + 2 AgNO3 ----> 2 AgCl + Ba(NO3)2

Question/Answer
10MillionJujubes asked on 04/08/07 - I need help on chemistry!

How many liters of CO2 gas are evolved when 2.00g of NaHCO3 react completely according to the equation NaHCO3(aq) + H+(aq) = H2O(l) + CO2(g) + Na+(aq)?
(given the density of CO2 gas is 1.961 g/l)

matrix answered on 04/08/07:

NaHCO3(aq) + H+(aq) ----> H2O(l) + CO2(g) + Na+(aq)?

2.00/84.0 = 0.024 moles of NaHCO3
This will produce 0.024 moles of CO2

1 mole of any gas is 22.4 liters at STP.
22.4 x 0.024 = 0.754 liters

10MillionJujubes rated this answer Excellent or Above Average Answer

Question/Answer
10MillionJujubes asked on 04/08/07 - Mass relations in Reactions

What mass in grams of sodium hydroxide is produced if 20.0 g of sodium metal are reacted with excess water according to the chemical equation 2Na(s) + 2H2O(l)?
2NaOH(aq) + H2(g)?

matrix answered on 04/08/07:

2 Na(s) + 2 H2O(l)----> 2 NaOH(aq) + H2(g)?

20.0/23.0 = 0.87 moles of sodium
This will produce 0.87 moles of NaOH

0.87 x 40.0 = 34.8 grams

10MillionJujubes rated this answer Excellent or Above Average Answer

Question/Answer
10MillionJujubes asked on 04/08/07 - HELP!Chemistry!

Your friend sits on a 25.0L ball, decreasing the volume to 17.5 L. What is the pressure inside the ball if the air pressure that day is 100 kPa?

matrix answered on 04/08/07:

P x V (initial) = P x V (final)

P (final) = P (initial) x [V(initial)/V(final)]
P = 100 x (25.0/17.5) = 143 kPa

10MillionJujubes rated this answer Excellent or Above Average Answer

Question/Answer
yakax007 asked on 04/05/07 - Net Ionic Help Needed!!!!

I have no clue what I am doing wrong here. I am 95% sure these are right but it keeps telling me they are wrong. PLEASE help!!!!

K2S + ZnSO4
... Zn^2+ + S^2- -> ZnS

CsOH + FeCl3
... Fe^3+ + 3OH^1- -> Fe(OH)3

FeSO4 + K3PO4
... 3Fe^2+ + 2PO4^-3 -> Fe3(PO4)2

matrix answered on 04/05/07:

K2S + ZnSO4
Zn^2+(aq) + S^2-(aq) -> ZnS(s)

CsOH + FeCl3
Fe^3+(aq) + 3 OH^1-(aq) -> Fe(OH)3(s)

FeSO4 + K3PO4
3 Fe^2+(aq) + 2 PO4^-3(aq) -> Fe3(PO4)2(s)

Rules for writing Net Ionic Equations
http://www.chem.vt.edu/RVGS/ACT/notes/Notes_on_Net_ionic_rxns.html

Question/Answer
weirdo asked on 04/03/07 - help!

calculate the number of grams of hydrogen in 4.50 mol H2SO4

matrix answered on 04/03/07:

4.50 moles of H2SO4 contains 4.50 moles of H2.

The molecular weight of H2 is 2.016.

4.50 x 2.016 = 9.07 grams

Question/Answer
sksara asked on 04/03/07 - I need help with Molarity ASAP Please!!!

Given the reaction:
Pb(NO3)2 + 2KCl = PbCl2 + 2KNO3

What is the molarity of 20.0 mL of KCl solution that reacts with 30.0 mL of 0.400 M Pb(NO3)2?

matrix answered on 04/03/07:

Pb(NO3)2 + 2 KCl ---> PbCl2 + 2 KNO3
From your equation you see that one Pb(NO3)2 will react with 2 KCl.
The normality of the Pb(NO3)2 solution is twice the molarity. It is 0.800 N

milliters x normality give milliequivalents

In titrations: At the equivalent point the number of milliequivalents on the left is equal to the number of milliequivalents on the right.

20.0 ml x X molarity = 30.0 ml x 0.800 N
20.0 X = 24.0
X = 1.20 M molarity of KCl

Question/Answer
VioletPepper asked on 03/24/07 - In Isotonic, Hypertonic nad Hypotonic solution.... does glucose matter?

A 0.9% NaCl (saline) and 5% glucose solutions are considered isotonic to red blood cells.

But what will happen to red blood cells placed in a 0.9% glucose solution?

matrix answered on 03/24/07:

A 0.9% NaCl (normal saline solution) and 5% glucose solutions are isotonic with body fluids.

Red blood cells placed in an isotonic solution will retain their normal size, that is they neither gain nor lose water by osmosis.

Question/Answer
stevieg asked on 03/22/07 - moles

Determine the moles for ammonium iodide in 5.810 grams

matrix answered on 03/22/07:

5.810 / 144.943 = 0.040087 ---> 0.0401 moles

stevieg rated this answer Bad/Wrong Answer

Question/Answer
stevieg asked on 03/22/07 - chem

A hygroscopic substance which absorbs water from the moist air and forms either a moist solid or solution. Which of the following compounds is hygroscopic?

a. silver(I) chloride, AgCl
b. sodium hydroxide, NaOH
c. calcium hydroxide, Ca(OH)2
d. znic chloride, ZnCl2

matrix answered on 03/22/07:

a. silver(I) chloride, AgCl ---- Not hygroscopic
b. sodium hydroxide, NaOH ---- Yes,it is hygroscopic
c. calcium hydroxide, Ca(OH)2 ---- Not hygroscopic
d. znic chloride, ZnCl2 ---- Yes,it is hygroscopic

stevieg rated this answer Excellent or Above Average Answer

Question/Answer
stevieg asked on 03/22/07 - chem

How many grams of copper(II) nitrate could be formed from the reaction of 1.823 g of copper with excess HNO3?

matrix answered on 03/22/07:

1.823 / 63.5456 = 0.02869 moles
0.02869 x 187.56 = 5.381 grams

stevieg rated this answer Bad/Wrong Answer

Question/Answer
stevieg asked on 03/22/07 - moles

Determine the number of moles of silver nitrate in 6.90 grams? round it off

matrix answered on 03/22/07:

6.90 / 169.87 = 0.0406 ---> 0.041 moles

stevieg rated this answer Excellent or Above Average Answer

Question/Answer
anyajerae asked on 03/20/07 - moles

How many moles of sulfur are present in 6.0 moles of Al2(SO4)3

matrix answered on 03/20/07:

One mole of Al2(SO4)3 contains 3 moles of S.
6 moles of Al2(SO4)3 will contain 18 moles of S.

Question/Answer
TrynaPass asked on 03/18/07 - Mole ques

Consider the reaction

CH4 + 2O2 ----> CO2 + 2H2O

Exactly 14.5 moles of CO2 are produced upon reaction of 34.6 moles of O2 and how many moles of CH4??

matrix answered on 03/18/07:

34.6 moles of O2 is enough to react with 17.3 moles of CH4 which would produce 17.3 moles of CO2. Since you formed 14.5 moles of CO2 this tells you that the O2 is present in excess.This also tells you that all of the CH4 present reacted and since you get one mole of CO2 for every every mole of CH4, you must have started with 17.3 moles of CH4.

Question/Answer
TrynaPass asked on 03/18/07 - Mole quest

For the reaction

2 S(s) + 3 O2(g) ----> 2 SO3(g)

how much SO3 will be produced from 2.0 mol O2 and excess S?



1. 2.0 mol SO3
2. Cannot be determined unless the limiting reactant is known.
3. 0.75 mol SO3
4. 1.3 mol SO3

matrix answered on 03/18/07:

With excess sulfur
3 O2----> 2 SO3

3/2 moles = 2/X moles
X = (2x2)/3 = 1.3 moles SO3

TrynaPass rated this answer Excellent or Above Average Answer

Question/Answer
TrynaPass asked on 03/18/07 - another mole quest

When the reaction P4O10 + KOH -----> K3PO4 + H2O
is balanced, how much KOH is required to react with 3 moles of P4O10??

matrix answered on 03/18/07:

P4O10 + 12 KOH ---> 4 K3PO4 + 6 H2O

1/3 moles = 12/X moles
X = 36 moles

TrynaPass rated this answer Excellent or Above Average Answer

Question/Answer
dalexi asked on 03/18/07 - mole question

Consider the reaction

CH4 + 2O2 ---> CO2 + 2H2O

Exactly 14.5 moles of CO2 are produced upon reaction of 34.6 moles of O2 and how many moles of CH4?

matrix answered on 03/18/07:

34.6 moles of O2 is enough to react with 17.3 moles of CH4 which would produce 17.3 moles of CO2. Since you formed 14.5 moles of CO2 this tells you that the O2 is present in excess.This also tells you that all of the CH4 present reacted and since you get one mole of CO2 for every every mole of CH4, you must have started with 17.3 moles of CH4.

dalexi rated this answer Excellent or Above Average Answer

Question/Answer
dalexi asked on 03/17/07 - mole question

burning 32g of ch4 in excess oxygen will produce how much co2 according to the equation:

CH4+2O2 ---> CO2+2H2O

a)4 mol
b)1 mol
c)2 mol
d)3 mol
e)32 mol

matrix answered on 03/17/07:

You have 2 moles of methane. This will give 2 moles of carbon dioxide.

dalexi rated this answer Excellent or Above Average Answer

Question/Answer
stevieg asked on 03/16/07 - Chemistry

number of moles of cobalt acetate in 4.88 grams?

matrix answered on 03/17/07:

Molecular weight of Co(OAc)2 is 177.02

4.88/177.02 = 0.0276 moles

stevieg rated this answer Excellent or Above Average Answer

Question/Answer
jannie asked on 03/16/07 - replacement

Which equation represents a double replacement reaction?

2 Na + 2 H2O 2 NaOH + H2
CaCO3 CaO + CO2
LiOH + HCl LiCl + H2O
CH4 + 2 O2 CO2 + 2 H2O

matrix answered on 03/16/07:

(1) 2 Na + 2 H2O---> 2 NaOH + H2
(2) CaCO3 ----> CaO + CO2
(3) LiOH + HCl ----> LiCl + H2O
(4) CH4 + 2 O2 ---> CO2 + 2 H2O

Rxn 1 is a single displacement or substitution reaction.

Rxn 2 is a decomposition reaction.

Rxn 3 is a double replacement reaction.

Rxn 4 is a combustion reaction.

Question/Answer
lisarob8 asked on 03/05/07 - net ionic equations

I'm having trouble writing the net ionic equation for LiBr(aq) + AgNO3(aq)

matrix answered on 03/05/07:

LiBr(aq) + AgNO3(aq)

TOTAL IONIC EQUATION
Li+(aq) + Br-(aq) + Ag+(aq) + NO3-(aq)---->Li+(aq) + NO3-(aq) + AgBr(s)

Subtract all ions that are the same on both sides of the arrow.

NET IONIC EQUATION
Br-(aq) + Ag+(aq) ---> AgBr(s)

lisarob8 rated this answer Excellent or Above Average Answer

Question/Answer
JustinK asked on 03/03/07 - Percent Yield, Please Help!

Given the balanced equation: 2KClO3 --- 2KCl+3O2

If the decomposition of 2.00g of KClO3 gives 0.720 of O2, what is the percent yield for the reaction?

(I dont know how to find the theoretical yield needed in order to solve this problem.)

matrix answered on 03/04/07:

You have 0.0163 moles of KClO3 and it made 0.0225 moles of O2.

From equation 2 moles of KClO3 gives 3 moles of O2.Then 0.0163 moles will give X moles.
2/1.0163 = 3/X
X = 0.0245 moles (Theory)

0.0225/0.0245 x 100 = % yield

Question/Answer
VioletPepper asked on 03/02/07 - Balanced Equation

What coefficient should O2 get to balanced the equation? And please why? what should be balanced? I dont get it, can I get an explanaition, thank you.

C5H8 + ? O2 ---> 5CO2 + 4H2O

matrix answered on 03/02/07:

C5H8 + 7 O2 ---> 5CO2 + 4H2O

You need to have the same number of each element on both sides of the arrow. You have 14 oxygen atoms on the right side of the arrow thus you need 14 oxygens on the left side.

VioletPepper rated this answer Excellent or Above Average Answer

Question/Answer
VioletPepper asked on 03/01/07 - Balanced Equations

What is the correct coefficient for each H2?

Al + H2SO4 ---> AL2(SO4)3 + ? H2
(2, 4, 1, 5, 3)

Fe + HCl ---> FeCL3 + ? H2
(1, 5, 3, 4, 2)

AND WHY?

matrix answered on 03/01/07:

2 Al + 3 H2SO4 ---> Al2(SO4)3 + 3 H2

2 Fe + 6 HCl ---> 2 FeCl3 + 3 H2

Be careful with your formulas. It is Al2(SO4)3 and not AL2(SO4)3 !
Also FeCl3 and NOT FeCL3.

VioletPepper rated this answer Excellent or Above Average Answer

Question/Answer
VioletPepper asked on 03/01/07 - Balanced equation in chemical reaction

How can I balanced this equation:

K3PO4 + Ca(NO3)2 ---> Ca3(PO4)2 + KNO3

matrix answered on 03/01/07:

2 K3PO4 + 3Ca(NO3)2 ---> Ca3(PO4)2 + 6 KNO3

Ellie33 rated this answer Excellent or Above Average Answer
VioletPepper rated this answer Excellent or Above Average Answer

Question/Answer
fifij asked on 02/13/07 - Mole/gram relationship

If you know the molecular weight of the molecule (in my case, 1 mole nitroglycerin = 227) how would you go about working out the products ratio when 10g of nitrolycerin is reacted?

matrix answered on 02/13/07:

The equation for nitroglycerin exploding is below.

4 C3H5(ONO2)3 ---> 12 CO2 + 10 H2O + 6 N2 + O2

10/227 = 0.044 moles

If you multiply each of the coefficients in the equation by 0.044 to obtain the new coefficients.

0.18 C3H5(ONO2)3 ---> 0.53 CO2 + 0.44 H2O + 0.26 N2 + 0.044 O2

Thus 0.18 moles of nitro gives 0.53 moles of CO2 ,0.44 moles of H2O , 0.26 moles of N2 and 0.044 moles of O2.

fifij rated this answer Excellent or Above Average Answer

Question/Answer
afgroppe asked on 02/08/07 - Compounds soluble in water...

Which compound or compounds in each of the following groups is (are) expected to be soluble in water?
(a) BaSO4, Ba(NO3)2, BaCO3
(b) Na2SO4, NaClO4, NaCH3CO2
(c) AgBr, KBr, Al2Br6

matrix answered on 02/08/07:

(a) BaSO4-Insoluble
Ba(NO3)2-Soluble
BaCO3-Insoluble

(b) Na2SO4-Soluble
NaClO4-Soluble
NaCH3CO2-Soluble
All sodium ionic compounds are soluble.

(c) AgBr-Insoluble
KBr-Soluble
Al2Br6-----Special---Aluminum bromide reacts violently with water. It is not dissolving but is instead being converted to new aluminum compounds.

See table on website below to learn about solubilities.

http://www.ausetute.com.au/solrules.html

Question/Answer
tstewart asked on 01/28/07 - help
help

Who many moles of carbon atoms are there in 0.500 moles of C2H6

matrix answered on 01/28/07:

There is one mole of carbon atoms in 0.500 moles of C2H6.

Question/Answer
ukballa asked on 01/25/07 - Chemisty Lab

Calculate the mass of HC2H3O2 in 1 L of a .500 M vinegar solution.

matrix answered on 01/25/07:

A 0.500 molar solution contains 0.500 moles of the solute in a liter of solution.
Your one liter of solution contains 0.500 moles of acetic acid.
The molecular weight of acetic acid (CH3CO2H) is 60.05.
0.500 X 60.05 = 30.0 grams

Question/Answer
boogieman asked on 01/23/07 - electronic configurations

what is the electronic configurations of:
the ground state of oxygen atom
the excited state of carbon atom
the ground state of nitrogen atom

matrix answered on 01/23/07:

Electronic configurations of:
(1)the ground state of oxygen atom
1s2 2s2 2p4

(2)the excited state of carbon atom
1s2 2s1 2p3

(3)the ground state of nitrogen atom
1s2 2s2 2p3

Question/Answer
boogieman asked on 01/23/07 - Help Orbitals

what is the shape of a dz2 orbital (that is d subscript z and superscript 2 on the z)

matrix answered on 01/23/07:

Hello Boogieman

See the following website for the shape of a dz2 orbital.

http://www.corrosionsource.com/handbook/periodic/e_orbits.htm

Question/Answer
fherna6 asked on 12/11/06 - Balancing the equation

What does the otherside of this equation look like?
NaOH+CH CO H ------> ?
3 2

matrix answered on 12/11/06:

This is a neutralization reaction. See website below for more examples.
http://chemistry.about.com/od/acidsbases/a/aa110204a.htm

NaOH + CH3CO2H ---> H2O + CH3CO2Na

Question/Answer
fherna6 asked on 12/11/06 - Balancing the equation

What does the otherside of this equation look like?
NaOH+CH CO H ------> ?
3 2

matrix answered on 12/11/06:

This is a neutralization reaction. See website below for more examples.
http://chemistry.about.com/od/acidsbases/a/aa110204a.htm

NaOH + CH3CO2H ---> H2O + CH3CO2Na

fherna6 rated this answer Above Average Answer

Question/Answer
CCSurferGirl asked on 12/05/06 - Molecular, Ionic, and Net Ionic Equations...

Hello!! I seem to still be having quite a bit of difficulty with these two problems...!!! If somebody could help, that would be great! Thanks So Much!!!

Write a Molecular, Ionic, and Net Ionic Equation for the following statement, and state what the oxidizing agent and reduction agent are.
A Solution of Nickel (II) bromide is added to a solution of potassium hydroxide.



Urea is a component of fertilizer. Urea provides all of the nitrogen that is present is fertilizer. If the fertilizer is rated as 6.00% nitrogen, how many grams of urea are present in 100.00 g of fertilizer?

matrix answered on 12/06/06:

MOLECULAR
NiBr2 + 2 KOH ---> Ni(OH)2 + 2 KBr

IONIC
Ni++(aq) + 2 Br-(aq) + 2 K+(aq) + 2 OH-(aq) ---> Ni(OH)2(s) + 2 K+(aq) + 2 Br-(aq)

NET IONIC
Ni++(aq) + 2 OH-(aq) ---> Ni(OH)2(s)

None of the reactants have a change in oxidation number. There is NOT a reducing or oxidizing agent in this reaction.

Question/Answer
CCSurferGirl asked on 12/05/06 - Two more Multiple Choice Questions...

Ok, Sorry I've got two more questions...These are multiple choice!! Thanks!!!!

According to the balanced equation, how many moles of HI would be necessary to produce 2.5 mol of I2, strting with 4.0 mol of KMnO4 and 3.0 mol of H2SO4?

10 HI + 2 KMnO4 + 3 H2SO4 --> 5 I2 + 2 MnSO4 + K2SO4 + 8 H2O

a. 20
b. 10
c. 8.0
d. 5.0
e. 2.5


A 1.0 L sample of an aqueous solution contains 0.10 mol of NaCl and 0.10 mol CaCl2. What is the minium number of moles of AgNo3 that must be added to the solution in order to precipitate all of the CL- as AgCl(s)?

a. 0.10 mol
b. 0.20 mol
c. 0.30 mol
d. 0.40 mol
e. 0.60 mol

matrix answered on 12/06/06:

10 HI + 2 KMnO4 + 3 H2SO4 --> 5 I2 + 2 MnSO4 + K2SO4 + 8 H2O
(d)5.0
From the equation you can see that need 10 moles of HI to make 5 moles of I2.Since you only need to make 2.5 moles of I2 then you only need 5 moles of HI. This is true IF you have enough of the other two reagents (you do have enough of the other reagents).





1.0 liter of 0.10 molar NaCl contains 0.10 moles of Cl-.
1.0 liter of 0.10 molar CaCl2 contains 0.20 moles of Cl-.
You need 0.30 moles of AgNO3.

Question/Answer
Phambrick88 asked on 12/04/06 - Chemistry help

How would you verify the limiting reactant of potassium Iodide and Lead (II) Nitrate using a calculation? You can assume there are 20 drops in 1 mL and you used 1 drop of 1.00M solutions of each reactant

matrix answered on 12/04/06:

2 KI + Pb(NO3)2 ---> PbI2 + 2 KNO3
(1/20) x 1.00M = 0.05 mmoles of each of the reagents.

You need 2 x 0.05 mmoles = 0.10 mmoles of KI to react with 0.05 mmoles of Pb(NO3)2. Since you only have 0.05 mmoles of KI, at the end of the reaction you will have 0.025 mmoles of Pb(NO3)2 remaining.
The KI is the limiting reagent.

Phambrick88 rated this answer Excellent or Above Average Answer

Question/Answer
erwyn asked on 12/03/06 - one more stoich problem, i promise

How many liters of O2 are needed to react completely with 45.0 L of H2S at STP?

matrix answered on 12/04/06:

2 H2S + 3 O2 ---> 2 H2O + 2 SO2

The volume of one mole of any gas at STP conditions is 22.414 liters.

45.0/22.414 = 2.01 moles of H2S
Since 2 moles of H2S requires 3 moles of O2 for complete oxidation then 2.01 moles will require 3.02 moles of O2.

2/3 = 2.01/X
X = (3 x 2.01)/2 = 3.02 moles of O2

3.02 x 22.414 = 67.4 liters

Question/Answer
cyclegirl asked on 12/02/06 - Balancing Redox Reactions

I'm trying to balance the following redox reaction:

H20 + K2S + KMnO4 --> S8 + KOH + MnO2

I presume that the original equation was:

K2S + KMnO4 --> S8 + MnO2

But, I'm completely stuck on where to go from here.

matrix answered on 12/02/06:

Hello cyclegirl
I have not worked on redox reactions for many years but I will give you my results. I am also including some websites to help you with further redox problems.

8 S-- ----> S8 + 16 electrons
3 electrons + MnO4- ----> MnO2

32 H2O + 16 KMnO4 + 24 K2S ---> 3 S8 + 16 MnO2 + 64 KOH




http://www.chemistrycoach.com/balancing_redox_in_acid.htm

http://www.mpcfaculty.net/mark_bishop/redox_balance_half_base_example.htm

http://library.kcc.hawaii.edu/external/chemistry/bal_equations_rules.html

http://www.ualberta.ca/~jplambec/che/p102/p02084.htm

http://notes.chem.usyd.edu.au/course/masters/Chem1/CHEM1102%20Directory/balancingredoxequations.htm

http://www.sparknotes.com/chemistry/electrochemistry/galvanic/section1.html

cyclegirl rated this answer Excellent or Above Average Answer

Question/Answer
deustice asked on 11/19/06 - mole question

how many grams of NH4CL and CaO are needed to make 0.100 mole of NH3?

matrix answered on 11/20/06:

Always write a correctly balanced equation to help you do these problems.
2 NH4Cl + CaO ---> 2 NH3 + H2O + CaCl2

The equation shows you need 2 moles of NH4Cl and one mole of CaO to make 2 moles of NH3.

Since you want 0.1 moles of NH3 I reduced the proportionally reduced the coefficients to give 0.1 moles of NH3.

0.1 NH4Cl + 0.05 CaO ---> 0.1 NH3 + 0.05 H2O + 0.05 CaCl2.

This shows you need 0.1 mole of NH4Cl and 0.05 moles of CaO.

0.1 x molecular weight of NH4Cl = grams NH4Cl
0.05 x molecular weight of CaO = grams CaO.

Question/Answer
bukola4 asked on 11/14/06 - help..net ionic equations

write a net ionic equation for the reaction resulting from mixing NaOH solution with Na2So4 solution


write a net ionic equation for the reaction resulting from mixing Na2S solution with Na2So4 solution

write a net ionic equation for the reaction resulting from mixing NaBr solution with NH4Cl solution

im really confused with these problems...all the rest i could find but these..would they be explained?

matrix answered on 11/15/06:

When all compounds on both sides of the arrow are soluble then there is NO net ionic equation. All of the ions are spectator ions and are not shown.

See solubility rules.
http://chemistry.che.georgiasouthern.edu/jlobue/Solubility%20rules.pdf

Question/Answer
lil_brat asked on 11/14/06 - sorry, to bother you. But i'm realy stuck with this question!

Inorgainc Complexes
1)Using the given mass of ferrous ammonium sulfate calculate the theoretical yield of K3[Fe(C2O4)3]*3H2O.

MAss of ferrous ammonium sulfate: 4.00g
(limiting reactant)

Mass of K3[Fe(C2O4)3]*3H2O: 5.00g

FOR K3[Fe(C2O4)3]*3H2O
2)What is the oxidation state of the central metal ion?
3)What is the coordination number of the central metal ion?

matrix answered on 11/15/06:

(1)The overall equation is below.

2 (NH4)2Fe(SO4)2 + 3 H2C2O4 + 4 H2O + H2O2 + 3 K2C2O4 -----> 2 K3[Fe(C2O4)3]�3H2O + 2 (NH4)2SO4 + 2 H2SO4

You have 4.00 g of (NH4)2Fe(SO4)2 and you indicated that this is the limiting reagent.
4.00/169.9 = 0.0235 moles of (NH4)2Fe(SO4)2

This should give 0.0235 moles of K3[Fe(C2O4)3]�3H2O
0.0235 x 490.8 = 11.5 g Theoretical yield
CHECK MY MATH!


(2)The Fe has oxidation state of +3.

(3)The coordination number of the Fe is 6.

lil_brat rated this answer Excellent or Above Average Answer

Question/Answer
squance asked on 10/28/06 - Combined Gas Law

Two samples of oxygen are available - one is 3 litres of gas at 1 atmosphere and 30 Celcius, the other is 4 litres of gas at 0.9 atmospheres adn 40 celcius. Which sample of oxygen contains the greater amount of gas? (Hint - convert both to the same conditions of temperature and pressure).

Use the P1xV1/T1 = P2V2/T2 rule where P=pressure, V=volume and T=temperature

matrix answered on 10/29/06:

(P1)(V1)/T1 = (P2)(V2)/T2
Solve for V2
V2 = (P1)(V1)(T2)/(T1)(P2)

For sample 1
V2 = (1 atm)(3 liters)(273 Kelvin)/(303 kelvin)(1 atm)
V2 = 2.7 liters

Do the same for sample 2.

This will show you the sample with the largest amount of gas.

squance rated this answer Above Average Answer

Question/Answer
Clavebear asked on 10/22/06 - Ionization

I need to know how to work the following problem.

In 1 mL of water there are 3 *10^22 molecules of H2O.

How many hydrogen ions are in 1 mL of water? (hint) In 1 billion water molecules only 2 are ionized.

matrix answered on 10/22/06:

How many times will one billion go into 3 x 10^22 ?????
One billion is 1 x 10^9.

(3 x 10^22)/(1 x 10^9) = 3 x 10^13
Since there are 2 molecules per billion then there are 2 x 3 x 10^13 = 6 x 10^13 molecules.

Clavebear rated this answer Excellent or Above Average Answer

Question/Answer
Chico asked on 10/15/06 - Need help

writing a molecular and ionic equation for the reaction between an aqueous solution of soduim hydroxide and sulfuric acid, and then calculating the number of moles of soduim hydroxide required to react with 25ml of the sulfuric acid solution

matrix answered on 10/17/06:

Molecular equation.
2 NaOH + H2SO4 ----> Na2SO4 + 2 H2O

Total ionic equation.
2 Na+(aq) + 2 OH-(aq) + 2 H+(aq) + SO4--(aq) ---> 2 Na+(aq) + SO4--(aq) + 2 H2O(l)

Net Ionic equation.
H+(aq) + OH-(aq) ---> H2O(l)
The Na+ and SO4-- ions are spectator ions and are NOT included in a net ionic equation.

You did NOT give the concentration of the H2SO4 solution!
Assuming it is pure H2SO4 then it is 18 molar.
25ml x 18M = 450 moles of H2SO4. Since it contains 2 H+ per molecule it will require 900 moles of NaOH.

Question/Answer
btrahan asked on 10/10/06 - net ionic equations

write a net ionic equation for the precipitation reaction resulting from mixing BaCl2 solution with K2So4 solution

matrix answered on 10/10/06:

Ba++(aq) + SO4--(aq) ---> BaSO4(s)

The Cl- and K+ ions are spectator ions and are not included in the net ionic equation.

Question/Answer
rippa125 asked on 09/17/06 - Moles of Al atom

I was wondering how to do this problem...


How many moles of Al atoms are needed to combine with 1.58 moles of Oxygen atoms to make aluminum oxide?

matrix answered on 09/17/06:

Always write the correct equation for your reaction to help you do the problem.

4 Al + 3 O2 ----> 2Al2O3
4/3 = X/1.58
X = (4 x 1.58)/3 = 2.11 moles of Al

rippa125 rated this answer Excellent or Above Average Answer

Question/Answer
5incere asked on 09/14/06 - methanol

explanation of why treating methanol poisoning with ethanol lowers the death rate.

matrix answered on 09/17/06:

Ethanol is a competitive substrate of alcohol dehydrogenase and thus prevents the metabolism of methanol.
Methanol is slowly converted to formaldehyde (by alcohol dehydrogenase) and then to formic acid in the liver.The formaldehyde is converted to formic acid by aldehyde dehydrogenase.Formic acid causes the metabolic acidosis and central nervous system damage seen in methanol poisoning.By giving ethanol by IV the metabolism of methanol is slowed.

5incere rated this answer Excellent or Above Average Answer

Question/Answer
SensationalBoy asked on 09/13/06 - ISOMERS OF C9H20.

I know there are suppose to be 35 Isomers for C9H20. I can find only a few. There are suppose to 1 nonane,3octane,11heptane,12hexane,8pentane. I have troubles finding some of the heptane, and hexane isomers. Please help me out.

matrix answered on 09/13/06:



35 Isomers of C9H20


n-Nonane
2-methyloctane
3-methyloctane
4-methyloctane
2,2-dimethylheptane
2,3-dimethylheptane
2,4-dimethylheptane
2,5-dimethylheptane
2,6-dimethylheptane
3,3-dimethylheptane
3,4-dimethylheptane
3,5-dimethylheptane
4,4-dimethylheptane
3-ethylheptane
4-ethylheptane
2,2,3-trimethylhexane
2,2,4-trimethylhexane
2,2,5-trimethylhexane
2,3,3-trimethylhexane
2,3,4-trimethylhexane
2,3,5-trimethylhexane
2,4,4-trimethylhexane
3,3,4-trimethylhexane
2-methyl-3-ethylhexane
2-methyl-4-ethylhexane
3-methyl-3-ethylhexane
3-methyl-4-ethylhexane
2,2,3,3-tetramethylpentane
2,2,3,4-tetramethylpentane
2,2,4,4-tetramethylpentane
2,3,3,4-tetramethylpentane
2,2-dimethyl-3-ethylpentane
2,3-dimethyl-3-ethylpentane
2,4-dimethyl-3-ethylpentane
3,3-diethylpentane

SensationalBoy rated this answer Excellent or Above Average Answer

Question/Answer
squance asked on 08/05/06 - Stoichiometry

What volume of 0.5M H2SO4 solution would react wtih 12.5 g of NaOH?
What mass of water would be formed by this reaction?
Could you please show the working out please?

matrix answered on 08/07/06:

12.5/40.0 = 0.3125 moles of NaOH

H2SO4 + 2 NaOH ---> Na2SO4 + 2 H2O
1 mole of H2SO4 will react with 2 moles of NaOH
X moles H2SO4 of will react with 0.3125 moles of NaOH
1/X = 2/0.3125
X = 0.3125/2 = 0.15625 This needs to be rounded to 3 places ---> 0.156 moles of H2SO4.
This is the amount of H2SO4 needed.
0.500M H2SO4 contains 0.500 moles of H2SO4 in 1000 ml of solution.
0.156/0.500 x 1000ml = 312 ml This is the amount of H2SO4 needed.
0.3125 moles of NaOH will react to give 0.3125 moles of H2O
0.3125 x 18.0 = 5.63g of H2O

squance rated this answer Excellent or Above Average Answer

Question/Answer
adharris3 asked on 07/02/06 - Net ionic equation

How do you write out the net ionic equation for this problem:

H2SO4(aq) + MgCO3(s) --> H2O(I) + CO2(g) + MgSO4(aq).

matrix answered on 07/03/06:

H2SO4(aq) + MgCO3(s) --> H2O(I) + CO2 (g) + MgSO4(aq).

NET IONIC EQUATION
2 H+(aq) + MgCO3(s) ---> Mg++(aq) + H2O(l) + CO2(g)

SO4-- is a spectator ion and is NOT included in the net ionic equation.

See sites below for help.
You will need to learn the solubility rules in order to do these equations.
http://www.towson.edu/~ladon/netionic.html
http://www.sparknotes.com/testprep/books/sat2/chemistry/chapter6section3.rhtml
http://www2.aacc.edu/sciljtracey/CHE111/c111helpionicequations.htm

adharris3 rated this answer Excellent or Above Average Answer

Question/Answer
sshayegh12 asked on 05/25/06 - Balancing Equations

What is the balanced equation for:

AlBr3 + Cl2 ---> AlCl3 + Br2

matrix answered on 05/25/06:

2 AlBr3 + 3 Cl2 ---> 2 AlCl3 + 3 Br2

sshayegh12 rated this answer Excellent or Above Average Answer

Question/Answer
aarietta asked on 05/19/06 - OCTET STRUCTURES

***SORRY I ACCIDENTLY POSTED IN THE MATH SECTION***

What exactly is an Octet Structure? and how is it more stable than others?


I had a hard time looking for a clear definition, but here is what I have thus far.

"Many stable molecules and polyatomic ions are composed of covalently bonded atoms having a noble-gas electron configuration and this configuration is referred to as the octet structures."

matrix answered on 05/22/06:

The octet rules states that when atoms form covalent bonds, they tend to share sufficient electrons so as to achieve an outer shell having eight electrons. An exception is hydrogen and helium, which have a stable outer shell of 2 electrons. The octet rule describes the attraction of elements towards having,WHENEVER POSSIBLE, eight valence-shell electrons (4 electron pairs) in their outer shell.A full outer shell with eight electrons is relatively stable,many atoms lose or gain electrons to obtain an electron configuration like that of the nearest noble gas.Except for Helium (it only has 2 electrons in the s orbitals), noble gases have eight electrons in their valence shells. A full shell of electrons in the outside electron ring is the most stable configuration. The first shell has only two electrons in a single s subshell. Helium has a full shell and is thus an inert element. Hydrogen, though, has only one electron. It can lose an electron to become H+, a hydrogen ion or it can gain an electron to become H-, a hydride ion. All the other shells have an s and a p subshell, giving them at least eight electrons on the outside. The s and p subshells often are the only valence electrons, thus the octet rule is named for the eight s and p electrons.

Question/Answer
crystal84 asked on 04/17/06 - Help

The balanced equation P4(s) + 6H2(g) �� 4PH3(g) tells us that 2 mol H2

a. More than 1.0 mol but less than 1.5 mol PH3

b. Roughly 0.5 mol PH3
c. Roughly 2.0 mol PH3
d. less than 0.5 mol PH3

matrix answered on 04/17/06:

P4(s) + 6H2(g) ---> 4PH3(g)

6 moles of H2 gives 4 moles of PH3
2 moles of H2 gives X moles of PH3
6/4 = 2/X
X = (2 x 4)/6 = 8/6 = 1.333 moles of PH3
Answer A: More than 1.0 but less than 1.5 moles

Question/Answer
crystal84 asked on 04/17/06 - Help

A 3.0-mol sample of KClO3 was decomposed according to the equation

2KClO3(s) �� 2KCl(s) + 3O2(g)

How many moles of O2 are formed assuming 100% yield?

a. 4.5 mol O2
b. 4.0 mol O2
c. 3.5 mol O2
d. 3.0 mol O2

matrix answered on 04/17/06:

2KClO3(s) ---> 2KCl(s) + 3O2(g)
2 moles KClO3 gives 3 moles of O2
3 moles KClO3 gives X moles of O2
2/3 = 3/X
X = (3 x 3)/2 = 4.5 moles

Question/Answer
crystal84 asked on 04/17/06 - help

An excess of Al and 6.0 mol of Br2 are reacted according to the equation

2Al + 3Br2 �� 2AlBr3

How many moles of AlBr3 will be formed assuming 100% yield?

a. 4.0
b. 3.0
c. 5.0
d. 6.0

matrix answered on 04/17/06:

2Al + 3Br2 ---> 2AlBr3
3 Moles of Br2 give 2 moles of AlBr3
6 moles of Br2 will give 4 moles of AlBr3

3/2 = 6/X
X = (6 x 2)/3 = 4

Question/Answer
crystal84 asked on 04/17/06 - help

For the reaction

C2H4(g) + 3O2(g) �� 2CO2(g) + 2H2O(g)

if 6.0 mol of CO2 are produced, how many moles of O2 were reacted?

a. 9.0
b. 6.0



c. 7.0
d. 10.0

matrix answered on 04/17/06:

C2H4(g) + 3O2(g) ----> 2CO2(g) + 2H2O(g)

From the equation you can see that 3 moles of O2 react to give 2 moles of CO2.
To form 6 moles of CO2 you need 9 moles of O2.

3/2 = X/6
X = (3x6)/2 = 9

Question/Answer
wayno123 asked on 04/10/06 - How many grams of silver can be recovered from 10.0 g of silver sulphide

How many grams of silver can be recovered from 10.0 g of silver sulphide?

matrix answered on 04/10/06:

Silver is 87.06% of silver sulfide (Ag2S).

Ag has atomic weight of 107.868
S has atomic weight of 32.064
Ag2S has molecular weight of 247.801

[(2 x 107.868)/247.801] x 100 = 87.06%

10.0g x .8706 = 8.71g silver

wayno123 rated this answer Excellent or Above Average Answer

Question/Answer
coolisha3 asked on 02/26/06 - Molarity Problem

Hi I can't seem to figure this one out.. please help...
If 4.00 mL of 0.0250 M CuSO4 is diluted to 10.0 mL with pure water, what is the molar concentration of copper(II) sulfate in the diluted solution?

Thanks

matrix answered on 02/26/06:

This is a very common problem.Use the following to solve.

ml x molarity (1st concentration) = ml x molarity(2nd concentration)

4.00 ml x 0.025M = 10.0 ml x X molarity
Solve for X
X = (4.00 x 0.025)/10.0 = 0.010M

Question/Answer
TOOLFan asked on 02/19/06 - URGENT HELP NEEDED

I am trying to complete this lab report for chemistry class on Tueday. My problem is:

The recipe for the volcanic eruption used in many science projects is the reaction
of bakin soda and vinegar. When these compounds are mixed together, the salt sodium acetate is formed as well as a gas that extinguishes a burning flame and a substance that turns blue cobalt chloride paper pink.

Here is the equation we must balance (I think)

NaHCO3 + C2H4O2 => H2 + H2O + NaC2H3O2

Is this the correct equation and can it be balanced?

matrix answered on 02/19/06:

NaHCO3 + HOAc ---> NaOAc + {H2CO3]
[H2CO3] ---> H2O + CO2

NaHCO3 + HOAc ---> NaOAc + H2O + CO2

Sodium bicarbonate reacts with acetic acid to form sodium acetate and carbonic acid.
Carbonic acid is unstable and quickly decomposes to water and carbon dioxde.
Carbon dioxide extinguishes the burning splint.


Cobalt chloride paper is used to test for the presence of water because it turns from blue to pink upon exposure to water.

CoCl2 + 6 H2O → CoCl2 � 6 H2O

TOOLFan rated this answer Excellent or Above Average Answer

Question/Answer
raezzoul asked on 02/18/06 - Stoichiometry:HELP!!

please help me!
the problem is the following...
The concentration of sulphuric acid in a car battery is of approximately 6 mol.dm-3. How can I find the amount in ml if the solution is of 35% H2SO4 and 65% H2O???

I need it as soon as possible...please and thank you..

matrix answered on 02/19/06:

35% by weight H2SO4 means there are 35 g of H2SO4 in 100 ml of solution.
The density of H2SO4 is 1.83 g/ml.
grams/density = volume
35 / 1.83 = 19.1 rounded to 19 ml

Also, battery acid is not 6 molar.
100 g of battery acid contains 35 g H2SO4 and 65 g H2O and has a volume of 84 ml [19 ml(H2SO4) and 65 ml(H2O)]. A liter of battery acid contains 417 g H2SO4.
417/98 = 4.3 M

Question/Answer
em-ei asked on 02/15/06 - elow.

1. In the biuret test for blood proteins, ecplain the function of the following reagents:
a.23% soduim sulfate
b. ether
c. copper sulfate and sodium hydroxide


2. What is the chemical composition of Millon's reagent? write the equation invovled in the reaction between the protein and the Millon's reagent.


- hope that you can help me..pls...

matrix answered on 02/15/06:

(1)
I have not seen a procedure for doing the biuret test that uses sodium sulfate or ether. Are you sure about these two materials.
In the biuret test divalent copper (CuSO4)reacts in alkaline solution (NaOH) with protein peptide bonds to form the characteristic purple- colored Biuret complex. A light blue color indicates a negative test, a purple-violet color a positive test.Materials containing two or more peptide bonds (three or more amino acids) form a purple-violet complex with copper salts in alkali solution. The nature of the color is probably due to the formation of a tetra-coordinated cupric ion (Cu+2 ) with amino groups.(See following site for structures).

(2)
In my answer to your earlier question I explained about Millon's reagent.
Millon's reagent is made by dissolving 1 part of mercury in 1 part of cold fuming nitric acid. This is diluted with twice its volume of water and decanting the clear solution after several hours. A postive Millon's test is given by any compound containing a phenolic hydroxy group. Any protein containing tyrosine will give a positive test of a pink to dark-red colour. This color is probably due to a mercury salt of nitrated tyrosine.(See site below for structures).

http://www.aug.edu/biology/Biuretproteinassay3.pdf

http://www.cerlabs.com/experiments/10875404480.pdf


Question/Answer
mcase asked on 02/05/06 - chemistry

a sample of alcohol has a density of 0.785g/ml. calculate the mass of 50.0 ml of the alcohol. calculate the volume of 50.g of the alcohol.

matrix answered on 02/05/06:

The density of a material is its mass divided by its volume.
Density = mass/volume
This equation can be rearranged to
mass = volume times density
and also as
volume = mass/densty.

mass = 0.785 times 50.0 = 39.25 = 39.3 grams

volume = 50.0/0.785 = 63.69 = 63.7 ml

mcase rated this answer Excellent or Above Average Answer

Question/Answer
XxSkyexX asked on 01/21/06 - Percent Yield

In a lab that I've recently done in my grade 11 chemistry class, I had to find the mass of copper which is formed when excess aluminum is reacted with a given mass of copper(11) chloride dihydrate. I've found the percent yield but I think my answer may be impossible. Instead of 100% I reached 142.5%. 8cm by 8cm aluminum foil, 2.00g copper(2) chloride dihydrate, 50 mL water.
3CuCl2 . 2H2O + 2Al -> 3 Cu + 2 AlCl3 + 6H2O, if I have 2 g of 3 CuCl2 . 2H2O, and the formula is
n= m/mm
n=2g/190.638 g/mol
n= 0.01049 mol

To find the mass of 3CuCl2 . 2H2O, do I add the 3 and the 2 in front of the compound and times 0.01049 mol by 3 or 5 or 6? I used 3 and my mass was 0.031473263 mol
m=n x mm
m= 0.031473263 mol x 63.546 g/mol
=2 g

My actual yield of copper was 2.85g, 163.15g of copper left over - 160.3g was the beakers weight = 2.85g of Cu
py= actual yield/expected yield x 100
=2.85g/2g x 100
=1.425g x 100
= 142.5%

Is that possible or did I make a mistake? Im not positive the materials I used were 100% pure, can that account for the large difference?
Please help, and thank you.

matrix answered on 01/21/06:

The molecular weight of copper(II) chloride dihydrate is 170.48.
3CuCl2*2H2O + 2Al -> 3 Cu + 2 AlCl3 + 6H2O
From your equation you can see that all of the copper in the copper(II) chloride dihydrate becomes metallic copper. The percent of copper in copper(II) chloride dihydrate is 37.274%.
Thus if you have 2.00 grams of copper(II) chloride dihydrate it contains 2.00 x 0.3727 = 0.754 grams of copper. This is the theoretical yield for your reaction.
If your copper weighed 2.85 grams, you have some other material present.
Did you wash your copper to remove the aluminum compounds from your product????
Do you have any unreacted aluminum present???
Did you dry your copper to remove all traces of water???

A handy website for problems is below.It will give atomic/molecular weights and percent compositions.
http://www.ch.cam.ac.uk/magnus/MolWeight.html

XxSkyexX rated this answer Excellent or Above Average Answer

Question/Answer
em-ei asked on 12/05/05 - test for phenols
test for phenols

hi...i need your help can you give me the chemical equations, visible effect,reagent, principles of ferric chloride test and millon's test...pls...

matrix answered on 12/08/05:

Ferric chloride test. Most phenols, enols and a number of other compounds with moderately acidic OH groups react with ferric chloride to give colored complexes.Iron(III) ions form strongly colored complexes with phenols. The color of the complexes vary from compound to compound. This test is designed to convert the weakly acidic phenols to their conjugate base which can then complex with ferric ion. If the phenol is water soluble, add a few drops of 2.5% aqueous ferric chloride solution to a 3% aqueous solution of the phenol. A deep red, green, or blue color is positive. If the phenol is not water soluble, dissolve 20 mg of the solid (or 1 drop of the liquid) in 1 mL of methylene chloride and add 1 drop of pyridine. Add 3 drops of 1% ferric chloride in methylene chloride. An intense color is a positive test. NOT ALL PHENOLS will give a positive test.
http://www.wellesley.edu/Chemistry/chem211lab/Orgo_Lab_Manual/Appendix/ClassificationTests/phenol_amine_nitro.html


Millon's reagent is made by dissolving 1 part of mercury in 1 part of cold fuming nitric acid. This is diluted with twice its volume of water and decanting the clear solution after several hours. A postive Millon's test is given by any compound containing a phenolic hydroxy group. Any protein containing tyrosine will give a positive test of a pink to dark-red colour. This color is probably due to a mercury salt of nitrated tyrosine.



Another test is the reaction of phenols with bromine.Phenols decolorize bromine solutions to form bromo dervatives.Aqueous bromine added to phenol forms a white precipitate (2,4,6-tribromophenol).

http://www.ucdsb.on.ca/tiss/stretton/chem2/orglab1.htm

em-ei rated this answer Excellent or Above Average Answer

Question/Answer
chemigal asked on 10/15/05 - Molecular Orbital Theory

Hi Matrix,

I'm having a difficult time trying to figure out where the extra electron bonding pair comes from in PCl6 -1 in hybridization. Seeing that P has a valence of 5 and can make sp3d1 hybridization, how does it go to sp3d2 hyrbridization when the atom itself has no extra electrons to bump up into the d sub shell.

Any insight into this would be appreciated.
- chemigal

matrix answered on 10/16/05:

PCl5 is a neutral compound.The bonding is 5 bonds (sp3d1) arranged as a trigonal bipyramid.
This material forms an ion (PCl6-) by adding a chlorine ion (Cl-). The additional electron causes a rehybridization to sp3d2 orbitals .These are arranged in octahedral geometry.

nneka rated this answer Excellent or Above Average Answer

Question/Answer
Carlita asked on 10/03/05 - Separation of the Components of a Mixture

How could you separate zinc chloride, ZnCl2, from zinc sulfide, ZnS?

matrix answered on 10/03/05:

ZnCl2 is soluble in water but ZnS is not soluble.

Stir mixture in water then filter.
The ZnS will be on the filter and the ZnCl2 will be in the water.

Carlita rated this answer Excellent or Above Average Answer

Question/Answer
nneka asked on 08/17/05 - heat of combustion

please matrix, can you hlep me with this question. l know l have to cancel the C2H4, and H2 on either side of the equation but after that l am lost. can you help me? thank you.

Calculate the heat of combustion for C2H6 from the
following information:

C2H4 + 3O2----2CO2 + 2H2O change of heat o/f ---1409.5KJ

C2H4 + H2------C2H2 change fo heat o/f---136.7KJ

H2 + 1/2O2-----H2O change of heat o/f---- 285.5KJ

matrix answered on 08/18/05:

(1) C2H4 + 3O2----2CO2 + 2H2O 1409.5kJ
(2) H2 + 1/2O2-----H2O 285.5kJ
(3) C2H4 + H2-----C2H6 136.7kJ

You want to add these three equations but equation (3)needs to be reversed.

C2H4 + 3O2----2CO2 + 2H2O ( -1409.5kJ)
H2 + 1/2O2-----H2O (-285.5kJ)
C2H6 ---> C2H4 + H2 (136.7kJ)

ADD THREE EQUATIONS.
C2H4 + 3 O2 + H2 + 1/2 O2 + C2H6---->
2 CO2 + 2 H2O + H2O + C2H4 + H2

Cancel terms and add were possible
7/2 O2 + C2H6 ---> 2 CO2 + 3 H2O

Now total heat terms
136.7 + (-1409.5) + (-285.5) = -1558.3kJ

nneka rated this answer Excellent or Above Average Answer

Question/Answer
nneka asked on 08/12/05 - pls help again

Explain why C20H40 is a solid at 25C, while C4H4 is a gas at 25C.

matrix answered on 08/13/05:

Larger molecules have a greater surface areas for van der Waals interactions. As a result, large molecules have stronger Van der Waals attractions between them and thus have higher boiling points. Conversely, smaller molecules have less surface area for Van der Waals interactions resulting in low boiling and melting points.



Please read the following website for an explanation of van der Waals attractions.

http://www.chemguide.co.uk/atoms/bonding/vdw.html

nneka rated this answer Excellent or Above Average Answer

Question/Answer
nneka asked on 08/12/05 - Melting point

Consider the following data:

Solubility in water(kg/kg H2O, 25C)

NaF----0.042
NaCI---0.0357
NaBr---1.16
Nai----1.84

Melting point(C)

NaF----988
NaCI---801
NaBr----755
Nai-----651


(a) explain the decreasing melting point from NaF to Nai.

(b) explain the increasing solubility in the same order

(c) Cesium fluoride has a melting point of 682C and a solubility of 3.67 kg/kg H2O at 25C. explain the diference between the properties of CsF and NaF.

matrix answered on 08/13/05:

Hello nneka

It is NaI , not Nai

(b) Solubility of ionic compounds is usually governed by its lattice energy. In general the higher the lattice energy, the lower the solubility.The lattice energy of a salt gives a rough indication of the solubility of the salt in water because it reflects the energy needed to separate the positive and negative ions in a salt.

(a) The greater the lattice energy the higher, the melting point of the compound.



Lattice energy
NaF: calculated: 910 kJ mol-1
NaCl: calculated: 769 kJ mol-1
NaBr: calculated: 732 kJ mol-1
NaI: calculated: 682 kJ mol-1

(c) Lattice energy of CsF:calculated: 735 kj mol-1

nneka rated this answer Excellent or Above Average Answer

Question/Answer
nneka asked on 08/12/05 - which is most or leasr polar

which of these bonds ia most polar and which is least: S--O, CI--CI, or CI--O? justify your arrangement.

matrix answered on 08/12/05:

Hello nneka

You need to be more careful with your symbols for the elements.
Do mean CI-CI or Cl-Cl ???

Bond polarity depends on the difference in electronegativities of the two elements sharing the bond.
S-O difference is 3.5-2.5 = 1.0 (Most polar)
Cl-Cl difference is 3.0-3.0 = 0 (Least polar)
Cl-O difference is 3.5-3.0 = 0.5


Here is a website with values for electronegativity.
http://www.ausetute.com.au/bondpola.html

nneka rated this answer Excellent or Above Average Answer

Question/Answer
Ccl471 asked on 08/06/05 - Organic Chemistry vs. Biochemistry

Dear Matrix:

Thank you for responding to my space shuttle descent question.

I saw in your profile that you have a PhD in Organic Chemistry. Is that another name for Biochemistry, or are they two different fields?



Many thanks,

C.L.

matrix answered on 08/06/05:

Hello C.L.

These are two different fields.

Organic chemistry is the scientific study of the structure, properties, composition, reactions, and synthesis of organic compounds. Organic compounds are composed of carbon and hydrogen, and can possibly contain any of the other elements such as nitrogen, oxygen, phosphorus, and sulfur.

Biochemistry is the study of the structure and interactions of the complex organic molecules found in LIVING SYSTEMS.
NOTE the last two words in the second definition (LIVING SYSTEMS).

I have NEVER had a biochemistry course.

Ccl471 rated this answer Excellent or Above Average Answer

Question/Answer
brooks101 asked on 08/01/05 - chemistry question

hello i have a question for you i was hoping u can help since ive already had asked some questions earlier in the day and you were so helpful with that question.
I am asking you these questions because i have an exam soon and i have questions but no answers so i wanna make sure i have the right answer and if not i wanna see the proper method which is why asking you has been so helpful
---------------------------------------------
heres my question
write a net ionic equation for each double displacement reaction in aqueous solution also identfy the spectator ions.
a) SnCl2 with K3PO4
b) NiCl2 with Na2CO3

-----------------------------------
2nd question
for the following complete the reactions and identify the conjugate acid base pairs
a)HPO4-2(aq) + OH(aq)
b)NH3(aq) + H3O+(aq)

-----------------------------------------
calculate the PH of each of the following solutions
a)[H3O+]=6.94x10 to the power of -8 moles/L
b)the solution that is formed by diluting 50ml of 0.4 mol/L HCl to a final volume of 1.0L


YOURE HELP IS GREATLY APPRECIATED IF YOU CAN HELP ME YOUDE BE MAKING MY DAY
THANK you thank you thank you

matrix answered on 08/02/05:

Hello Brooks
I answer questions whenever I get some extra time. Because of this I only answer one problem per question.

a)SnCl2 with K3PO4

IONIC EQUATION
3 Sn++(aq) + 6 Cl-(aq) + 6 K+(aq) + 2 PO4---(aq) ----> Sn3(PO4)2(s) + 6 Cl-(aq) + 6 K+(aq)
Cancel terms that appear on both sides of the equation. These are the spectator ions.
NET IONIC EQUATION
3 Sn++(aq) + 2 PO4---(aq) ---> Sn3(PO4)2(s)


b) NiCl2 with Na2CO3
Ni++(aq) + 2 Cl-(aq) + 2 Na+(aq) + CO3--(aq) ---> NiCO3(s) + 2 Na+(aq) + 2 Cl-(aq)
NET IONIC EQUATION
Ni++(aq) + CO3--(aq) ---> NiCO3(s)

Question/Answer
brooks101 asked on 08/01/05 - chemistry question

hello,
i have a question for you I would appreciate it if u can give me some help with or direct me to where I can get help with this
two solutions are mixed together 2.5g of NaNO3 dissolved in 100ml of water and 17.7g of KNO3 dissolved in 80ml of water calculate the concentration of each major ion(Na+, K+, and NO3) in the mixture

matrix answered on 08/01/05:

Hello Brooks

NaNO3: 2.5/84.98 = 0.029 moles
KNO3: 17.7/101.1 = 0.175 moles

Na+: 0.029 moles
K+: 0.175 moles
NO3-: 0.029 + 0.175 = 0.204 moles

Molarity is moles /liter

Na+: 0.029/0.180liters =
K+: 0.175/0.180 =
NO3-: 0.204/0.180 =

brooks101 rated this answer Excellent or Above Average Answer

Question/Answer
nneka asked on 07/19/05 - chemistry 12

calculate the mass of potassium chlorate needed to prepare 5.00L of oxygen gas at 24.0C and 0.950atm(when using atmospheres, the gas constant, R=0.0821L.atm.mol-7.K-1

The equation is KCIO3----KCI+3/2O2

2. consider the following reaction, which takes place in an autoclave at 250C and 800atm.

NH3(g)+ 7/4O2(g)----NO2(g)+ 3/2H2O

into the reaction vessel has been placed 200L of NH3(g) and 120L of O2(g). The reaction is allowed to go to completion. determine the quantity, in moles, of the gas that remain unreacted.

matrix answered on 07/20/05:

Calculate the mass of potassium chlorate needed to prepare 5.00L of oxygen gas at 24.0C and 0.950atm(when using atmospheres, the gas constant, R=0.0821L.atm.mol-7.K-1
2 KCIO3 ---> 2 KCI + 3 O2

PV = nRT
n = PV/RT = (0.950)(5.00)/(0.0821)(297) = 0.195 moles

From equation 2 moles of KClO3 gives 3 moles of O2
2/3 = X/0.195
X = (2)(0.195)/3 = 0.130 moles of KClO3

M.Wt. KClO3 is 122.5
122.5 x 0.130 = 15.9 grams

nneka rated this answer Excellent or Above Average Answer

Question/Answer
nneka asked on 07/19/05 - chemistry 12

Consider the following reactions, which takes place in an autoclave at 250C and 800atm.
NH3(g)+7/4O2(g)-----NO2(g)+3/2H2O(g)

Into the reaction vessel has been placed 200L of NH3(g) and 120L of O2(g). the reaction is allowed to go to completion. determine the quantity, in moles, of the gas that remains unreacted.

2. A chemist wished to prepare 200mL of a saturated solution of BaF2, the concentration of which is 6.3x10-3 mol/L. determine the mass of BaF2 that will be needed to make up the 200mL of the solution.

matrix answered on 07/19/05:

Hello nneka

I will help with some of your problems BUT only one problem per question!

A chemist wished to prepare 200mL of a saturated solution of BaF2, the concentration of which is 6.3x10-3 mol/L.
Determine the mass of BaF2 that will be needed to make up the 200mL of the solution.

BaF2.... 175.32g/mole
175.32 x 6.3x10-3 = 1.1 g/L
One liter of saturated solution contains 1.1 grams of BaF2.
200mL will contain 1.1 x 200/1000 = 0.22 grams

nneka rated this answer Excellent or Above Average Answer

Question/Answer
nneka asked on 07/19/05 - chemistry

please can someone help me with this. l am rather stuck.
find the volume of gas at -30C and 0.75atm if it occupies a volume of 255L and 1.25atm

matrix answered on 07/19/05:

You did not give the temperature of the gas at the new volume so I will assume it is still -30C.

PV/nRT (1st set of conditions)= PV/nRT (2nd set of conditions)
Since n,R and T are the same for each side of equation,the equation becomes
PV = PV
This is Boyle's Law

X liters(0.75) = 255(1.25)
X = 255(1.25)/0.75 = 425 liters

nneka rated this answer Excellent or Above Average Answer

Question/Answer
squall29 asked on 06/28/05 - calculation of amount of BaCl & percent yield

KAl(so4)12H2O(aq)+2BaCl2(s)-> KCl+AlCl2+2BaSO4(s)
in this equation, what is the amount of BaCl needed to reack w/ a 25mL of a 0.10M alum solution?
and what is the % yield of BaSO4 if 1.02g is isolated?

matrix answered on 06/29/05:

Be careful with your formulas.You made a number of errors in the formulas in your question.

KAl(SO4).12H2O(aq) + 2BaCl2(aq)--> KCl(aq) + AlCl3(aq) + 2 BaSO4(s)

What is the amount of BaCl2 needed to react with 25mL of a 0.10M alum solution?

25 x 0.1M = 2.5 mmoles of alum
From equation you can see that 1 mmole of alum will react with two mmoles of BaCl2.
2.5 mmoles of alum will react with 5 mmoles of BaCl2.
A mmole of BaCl2 is 207.8 mg.
5 mmoles of BaCl2 is 1039mg or 1.04 g.




What is the % yield of BaSO4 if 1.02g is isolated?

You should make 5 mmoles of BaSO4. A mmole of BaSO4 is 233.9mg
5 mmoles is 1170mg or 1.17g
1.02/1.17 x 100 = 87.2 %yield.

Question/Answer
hollyv asked on 06/28/05 - Balanced Acid-Base Equation

What is the Balanced equation for c2h4o2 + NaOH in a titration neutralization?

matrix answered on 06/28/05:

C2H4O2 + NaOH ----> NaC2H3O2 + H2O

Question/Answer
bentz4 asked on 06/25/05 - ph level

What is the ph level of 1.0 x 10-3 M hydrochloric acid?

matrix answered on 06/26/05:

For a hydrochloric acid concentration of 1.0 x 10-3 M
the pH = -log [1.0 x 10-3] = -(-3.00) = 3.00

bentz4 rated this answer Excellent or Above Average Answer
nneka rated this answer Excellent or Above Average Answer

Question/Answer
bentz4 asked on 06/25/05 - equation

How many grams of solute are in 53 ml of 1.5M NaOH?

matrix answered on 06/25/05:

53 ml x 1.5M = 79.5 mmoles of NaOH
A mmole of NaOH is 40 mg

79.5 x 40 = 3180 mg = 3.180 g
This should be rounded to 3.2 g

bentz4 rated this answer Excellent or Above Average Answer

Question/Answer
snuffy296 asked on 06/05/05 - Sodium hydroxide

Show a chemical equation for the industrial production of sodium hydroxide.

matrix answered on 06/06/05:

Sodium hydroxide is producted by the electrolysis of a solution of sodium chloride.

2 Na+(aq) + 2 Cl�(aq) + 2 H2O(l)--->Cl2(g) + H2(g) + 2 Na+(aq) + 2 OH�(aq)


http://www.iupac.org/didac/Didac%20Eng/Didac03/Content/R18%20-%20R19.htm
http://scifun.chem.wisc.edu/chemweek/Cl2&NaOH/Cl2&NaOH.html

snuffy296 rated this answer Excellent or Above Average Answer

Question/Answer
erica2323 asked on 05/06/05 - Stoichiometry--need to translate your responses into DImensional Analysis

Hi-

I asked three Stoich questions yesterday and you responded to two of them. Raridon to all three-however, We are taught to use something like this:

Mass Given x Mol Given/ x Mol of Wanted/ x Mass Wtd/
(in Problem) Mass Given Mol Given Mol Wtd
(mole ratio)
I need to know how to solve the questions I asked using this. I can't translate your responses into this.... Dimensional Analysis. Thanks!

matrix answered on 05/09/05:

You need to understand the steps in doing reactions and not merely plugging numbers into a magic formula. My answers are consistent with dimensional analylis procedures.

First, always write a correctly balanced equation for your problem.The numbers in front of the chemical formulas tell you the number of moles required of each reagent.
For example in one of your problems,
CH4 + 2 O2 ----> CO2 + 2 H2O
This tells you that if you burn one mole of methane you will obtain one mole of carbon dioxide and two moles of water and will require two moles of oxygen.
In your problem you obtained 25.0 g of CO2.
This is 25.0g/44.0g/mole = 0.568 moles of CO2.
Since one mole CH4 gives one mole of CO2,then to make 0.568 mole of CO2 you need 0.568 mole of methane.
0.568mole x 16.0g/mole = 9.09g of methane

Also from equation you know that the reaction requires two moles of oxygen for every one mole of methane.
0.568 x 2 = 1.136 moles of oxygen are needed.
1.136 moles x 32.0 g/mole = 36.4 g of oxygen.

Question/Answer
erica2323 asked on 05/05/05 - Stoichiometry Excess Reactants Questions

I need to SEE the steps to solve these types of questions...

How much AgCl is produced if 100 grams of BaCl2 react with excess AgNO3?

matrix answered on 05/05/05:

WRITE EQUATION !

BaCl2 + 2 AgNO3 ---> Ba(NO3)2 + 2 AgCl

100 g BaCl2 is 0.480 moles. This will give 0.960 moles of Cl ion.
100/208.2 = 0.480 moles BaCl2
0.480 x 2 = 0.960 moles Cl

0.960 moles of Cl will react with excess Ag+ to form 0.960 moles of AgCl.
0.960 x 143.3 = 137.6 g of AgCl. This should be rounded to 138 g AgCl

Question/Answer
erica2323 asked on 05/05/05 - Stoichiometry-Minimum amount needed of reactants

Can someone show all the steps-PLEASE??

In the reaction between CH4 and O2, if 25.0 g of CO2 are produced, what is the minimum amount of each reactant needed?

matrix answered on 05/05/05:

Always write a balanced equation for your reaction.
CH4 + 2 O2 ----> CO2 + 2 H2O

One mole of methane gives one mole of CO2.

25.0 g of CO2 is 0.568 moles.
You need 0.568 moles of methane to make 0.568 moles of CO2.
0.568 x 16.0 = 9.09 g of methane.

From equation, you can see it takes two moles of O2 for each mole of methane.

0.568 x2 x 32.0 = 36.4 g of O2.

Question/Answer
crimsonkristin asked on 01/04/05 - Substitution and Addition Reactions

Are these substituion or addition reactions? How can you tell and determine this? What are the products?

trichloromethane + chlorine -->
propene + bromine -->
ethylene + hydrogen iodide -->
ethane + chlorine -->

matrix answered on 01/04/05:


SUBSTITUTION: The compounds are saturated.If you are to have a reaction then something has to be substituted by your other reactant.In these two cases you are substituting a chlorine for a hydrogen.
CHCl3 + Cl2 ---> CCl4 + HCl

CH3CH3 + Cl2 ---> CH3CH2Cl + HCl ....This can continue until you have CCl3CCl3.It depends on how much chlorine you have.
http://www.chemguide.co.uk/mechanisms/freerad/multisubcl.html



ADDITION: You are adding to a double bond.
CH3CH=CH2 + Br2 ---> CH3CHBrCH2Br
CH2=CH2 + HI ---> CH3CH2I


crimsonkristin rated this answer Above Average Answer

Question/Answer
crimsonkristin asked on 12/23/04 - titration analysis of vinegar

Several 10.00mL samples of acetic acid are titrated with a standard 0.202mol/L solution of NaOH(aq). Phenolphthalein indicator, which changes from colorless to pink at the endpoint of the reaction, is used to detect the equivalence point.
What is the concentration of acetic acid, CH3COOH(aq, in a sample of vinegar?

matrix answered on 12/25/04:

I will do one of the four. You need to do the other three calculations then average the four values and round off to three places.

mmoles of acetic acid = mmoles of NaOH
ml x M = ml x M
10.00ml x X = 13.7ml x 0.202M
X = (13.7 x 0.202)/10.00
X = 0.277M

crimsonkristin rated this answer Excellent or Above Average Answer

Question/Answer
denberg asked on 12/20/04 - The chemistry of life


Michael Drosnin's The Bible Code 2: The Countdown, claims Francis Crick, who discovered DNA in 1953, believes it was sent here in a spaceship by aliens. Was Crick misquoted?

matrix answered on 12/20/04:

Hello Paul

I had never heard this but some searching gave the following.
Crick and Leslie Orgel proposed the theory of directed panspermia.
Leslie Orgel is now at Salk Institute for Biological Studies in San Diego but earlier he contributed to the development of ligand- field theory and wrote a textbook that I used in graduate school.
[source: Crick, F. H. C., and Orgel, L. E. "Directed Panspermia," Icarus, 19, 341 (1973)].

Here are some other sources that discuss panspermia.
http://en.wikipedia.org/wiki/Panspermia#Directed_Panspermia
http://www.essays.cc/free_essays/a2/svx44.shtml
http://www.daviddarling.info/encyclopedia/D/dirpans.html
http://www.time.com/time/archive/preview/0,10987,1101030217-421016,00.html

denberg rated this answer Excellent or Above Average Answer

Question/Answer
crimsonkristin asked on 12/04/04 - finding products

I was given a question to solve for what the molar concentration of silver nitrate in the solution to be recycled is and I'm trying to set up the chemical formula for this problem from the data I've been given which I've ommited here which I hope you don't need to help me with this. So far I have,
AgNO3(aq) + Na2SO4(aq) -->
for my reactants and I'm not sure how to get the products?

matrix answered on 12/05/04:

Hello crimisonkristin

You need to learn how to recognize reaction types. This is an example of a double replacement reaction.
Two compounds exchange parts in solution to form two new compounds, often (not always) with one of the new compounds being insoluble.

A solution of silver nitrate reacts with a solution of sodium sulfate to form silver sulfate and sodium nitrate.

2 AgNO3 + Na2SO4 ---> Ag2SO4 + 2 NaNO3

See sites below.
http://www.cameron.k12.wi.us/high/chemistry/drreac.html
http://www.spx.org/faculty/fred/chemistry/double_replacement_rxn.html
http://web.fccj.org/~smilczan/Two5/DR.html

crimsonkristin rated this answer Excellent or Above Average Answer

Question/Answer
crimsonkristin asked on 12/04/04 - Solvay process

What is the net reaction for the Solvay process? How do you figure it out?

matrix answered on 12/04/04:

step(1): CaCO3 + (heat)---> CaO + CO2

step(2): NH3 + CO2 + NaCl + H2O ---> NaHCO3 + NH4Cl

(all components are water-soluble)

Sodium bicarbonate converts to sodium carbonate on heating:

step(3): 2 NaHCO3 + (heat)--> Na2CO3 + H2O + CO2

The CO2 released is recycled to the previous step

The ammonia is recycled:

step(4): 2 NH4Cl + CaO ---> 2 NH3 + H2O + CaCl2

After all the recycling, the overall conversion is the simple exchange reaction:

overall: 2 NaCl + CaCO3 ---> Na2CO3 + CaCl2




Add all the reactions in steps 1-4.(You need to multiply all terms in step 2 by 2). Next subtract any terms that are on both sides of the arrow. This will leave the final overall equation.

crimsonkristin rated this answer Excellent or Above Average Answer

Question/Answer
bluedicius asked on 11/25/04 - predicting mass

2.73g of aluminum reacts in a single replacement reaction with excess sulfuric acid, what mass of hydrogen gas is produced from this reaction?

matrix answered on 11/30/04:


2 Al + 3 H2SO4 ----> 3 H2 + Al2(SO4)3

2.73/26.98 = 0.101 moles of Al
From equation you can see that for every two moles of Al you form three moles of H2.
2/3 = 0.101/X
X = 0.152 moles of H2
0.152 x 2.02 = 0.307 grams of H2

bluedicius rated this answer Excellent or Above Average Answer

Question/Answer
denberg asked on 11/24/04 - The De-Salinisation process...



I've often read articles in encyclopaedias which say that in the 70s there was a proposal to tow icebergs from the South Pole to Saudi Arabia, so that they could be melted there to provide fresh drinking water. The bergs would be covered in a special shroud during the journey, to minimize melting en route. This was thought to be a possible solution to Saudi Arabia's serious water shortage. The reasoning behind it was that it would be cheaper to tow the bergs than to desalinate the sea water along the Saudi coast.

Now, of course, 30 years later we all know that this plan was never put into practice. However, one thing puzzles me: If icebergs are made from frozen seawater, then wouldn't they be full of salt and therefore have to be desalinated, just like sea water from the Persian gulf? And wouldn't that defeat the whole object?

The articles I read never mentioned that, so what is
it that I've misunderstood here?

Many thanks,

Paul.

matrix answered on 11/24/04:

Hello Paul

Icebergs are NOT made of frozen seawater. They are formed when glaciers on the land reach the sea and break off and fall in.
Glaciers are formed from compacted snow. They are actually fairly pure water.

denberg rated this answer Excellent or Above Average Answer

Question/Answer
denberg asked on 11/24/04 - Heavy water



Would it be safe to drink heavy water - the water that is used to make an atom bomb?

Many thanks,

Paul.

matrix answered on 11/24/04:

Hello Paul

It is toxic if you drink a LARGE amount of it. You would have problems if you replace approximately half of the water in your body with heavy water. This would be an expensive process. I pay $160/100grams of 100%D deuterium oxide. It would take many 1000's of grams of D2O to replace half of the water in your body.

Here are some sites that explain the chemistry of this toxicity.
http://www.straightdope.com/mailbag/mheavywater.html

http://yarchive.net/med/heavy_water.html

denberg rated this answer Excellent or Above Average Answer

Question/Answer
crimsonkristin asked on 11/23/04 - chemical analysis

A chemical technician analyzed a sample of a waste solution and reported the findings to environmental chemists monitoring the lead content of the waste. The sample was mixed with excess potassium bromide so that any lead(II) nitrate in the solution would react; 3.65g of precipitate was produced. What is the mass of lead(II) nitrate in the sample?

matrix answered on 11/24/04:

Hello crimsomkristin

You need to do some of the work for your questions. You should give equations, molecular weights,etc, and if possible how you think the problem is worked.
Also,only one problem per question.

Pb(NO3)2 + 2 KBr ---> PbBr2 + 2 KNO3
3.65/367.0 = 0.00995 moles of PbBr2

0.00995 moles of Pb(NO3)2 is
0.00995 x 331.2 = 3.30 g

crimsonkristin rated this answer Excellent or Above Average Answer

Question/Answer
crimsonkristin asked on 11/22/04 - equation

Calculate the mass of lead(II) chloride precipitate produced when 2.57g of sodium chloride in solution reacts in a double replacement reaction with excess aqeous lead(II) nitrate.

matrix answered on 11/22/04:

2 NaCl + Pb(NO3)2 ---> 2 NaNO3 + PbCl2
2.57/58.44 = 0.0440 moles

0.0440 moles of NaCl will form 0.0220 moles of PbCl2
0.0220 x 278.1 = 6.12 g of PbCl2

crimsonkristin rated this answer Excellent or Above Average Answer

Question/Answer
crimsonkristin asked on 11/22/04 - gravimetric stoichiometry

Powerdered zinc metal reacts violently with S8 when heated. Predict the mass of sulfur required to react with 25g of zinc.

What formula do you use and whats the answer?

matrix answered on 11/22/04:


Zn + S ---> ZnS

25/65.37 = 0.38 moles of Zn
You need 0.38 moles of S for reaction.
32.06 x 0.38 = 12 g of S

Look at the equation for the answer to your question.
If you use S8 in your equation then the following would be true.
8 Zn + S8 ----> 8 ZnS
You would get the same answer as above.

crimsonkristin rated this answer Excellent or Above Average Answer

Question/Answer
Librose asked on 11/21/04 - VSEPR/Hybirdization/diagrams HELP!!

Sorry this is so long but I don't get any of it.

For these 2 molecules:
C2H4
NH2OH

1. Using VSEPR, how to u predict the shape?
2. Indicate hybridization of the central atom???
3. Sketch 3-D diagram an indicate bond angles
4. Demonstrate concept of hybirdization using the molecule H2CO3. Show electron configuration, hybird orbitals, hybird orbitals, a complete diagram, and sigma and pi bond (if any)

matrix answered on 11/21/04:

I do not have time to answer your questions in detail but the following websites will help you.The first two will have the answers to most of your questions.

http://www.chem.hawaii.edu/Bil301/classes/chapter10.ppt

http://cwx.prenhall.com/bookbind/pubbooks/hillchem3/medialib/media_portfolio/10.html

http://www.chemistry.ohio-state.edu/~grandinetti/teaching/Chem121/lectures/VSEPR/VSEPR.html

http://www.science.uwaterloo.ca/~cchieh/cact/c120/vsepr.html

Question/Answer
crimsonkristin asked on 11/19/04 - reactions

What are the balanced chemical reactions and types of reactions for these:

Carbon monoxide and iron(III) oxide react to produce iron and carbon dioxide.

Aluminum and copper(II) sulphate react to produce copper and aluminum sulphate.

Iron(III) nitrate and lithium hydroxide react to produce iron(III) hydroxide and lithium nitrate.

Sodium reacts with oxygen to produce sodium oxide.

Propane (C3H8) reacts with oxygen.

Mercury(II) oxide produces mercury and oxygen when heated.

matrix answered on 11/20/04:

Fe2O3 + 3 CO -----> 2 Fe + 3 CO2
Oxidation and reduction

2 Al + 3 CuSO4 ---> 3 Cu + Al2(SO4)3
Single replacement

Fe(NO3)3 + 3 LiOH ---> Fe(OH)3 + 3 LiNO3
Double replacement

4 Na + O2 --> 2 Na2O
Synthesis reaction

C3H8 + 5 O2 ---> 3 CO2 + 4 H2O
Combustion

2 HgO ----> 2 Hg + O2
Decomposition


http://www.chemtutor.com/react.htm

crimsonkristin rated this answer Excellent or Above Average Answer
lapaix4mum rated this answer Excellent or Above Average Answer

Question/Answer
Librose asked on 11/20/04 - Pedict lab HELP

Can you please keep this private.

What does sodium chloride do when it is added to the crystals of polymethylacrylate and water?

Why does acetone eat away at a styrofoam cup

And I also have to predict what will happen in these labs and anwser the questions (I have no clue how to anwser them).

http://img104.exs.cx/my.php?loc=img104ℑ=Predictthis8.jpg

and this

http://img104.exs.cx/my.php?loc=img104ℑ=Predictthis9.jpg


Sorry the labs take long to open

Thank you

matrix answered on 11/20/04:

I cannot go to your websites. I go to the imageshack home page but it will not go to your sites.

Acetone is an excellent solvent for polystyrene.The cup is dissolving.


I need more details for the first question about polymethylacrylate and water.

Question/Answer
crimsonkristin asked on 11/18/04 - moles

If 2.0 moles of nitroglycerin explodes, what is the number of moles of gaseous products are produced?

What is the number moles of Al(l) produced if 4.5 moles of Al2O3(s) reacts?

How many moles of C(s) is necessary to produce 3.2 moles of Al(l)?

matrix answered on 11/18/04:

The balanced equation for nitroglycerin explodng is below. It shows that 4 moles of nitroglycerin gives 29 moles of gaseous products. Two moles will give 14.5 moles of gaseous products.

4 C3H5N3O9(l)--->12 CO2(g)+6 N2(g)+O2(g)+10 H2O(g)

crimsonkristin rated this answer Excellent or Above Average Answer

Question/Answer
crimsonkristin asked on 11/18/04 - precipitation equation

What balanced equation do you get when you add colorless sodium hydroxide to blue copper (II) nitrate to produce a light blue precipitate?

matrix answered on 11/18/04:

Cu(NO3)2(aq) + 2 NaOH(aq) ==> Cu(OH)2(s) + 2 NaNO3(aq)

crimsonkristin rated this answer Excellent or Above Average Answer

Question/Answer
crimsonkristin asked on 10/31/04 - concentration of a solution

The brine solution in a home water-softening system has a salt concentration of 25% W/V. What mass of salt is dissolved if the brine tank holds 50L of solution?

matrix answered on 10/31/04:

A 25% w/v solution contains 25 grams of solute in 100 ml of solution.

This is 250 grams/liter.

50 x 250 = 12,500 grams in 50 liters.

crimsonkristin rated this answer Excellent or Above Average Answer

Question/Answer
steam asked on 10/20/04 - question 6

If excess sulfuric acid reacts with 30.0 g. of sodium chloride, how many grams of hydrogen chloride are produced?

matrix answered on 10/20/04:



H2SO4 + 2 NaCl ----> 2 HCl + Na2SO4

Calculate the number of moles of NaCl.This will also be the number of moles of HCl made.
30.0/58.4 = 0.513 moles

Number of moles X molecular weight of HCl
36.5 x 0.513 = 18.7 grams HCl

steam rated this answer Excellent or Above Average Answer

Question/Answer
crimsonkristin asked on 10/14/04 - solubility and conductivity

Is NaCl, table salt, a conductor when in solution? Is powdered magnesium soluble?

matrix answered on 10/14/04:

When sodium chloride is dissolved in water ,the solution is an excellent conductor of electricity.


Is powdered magnesium soluble?
Soluble in WHAT ????
You need to specify the solvent.
Powdered magnesium is not soluble in water.

crimsonkristin rated this answer Excellent or Above Average Answer

Question/Answer
steam asked on 10/13/04 - question 4

How many grams of copper are required to replace 4.00 g. of silver nitrate which are dissolved in water?

matrix answered on 10/13/04:


You have 0.0235 moles of AgNO3
4.00/169.87 = 0.0235

Cu + 2 AgNO3 ---> Cu(NO3)2 + 2 Ag

From equation you can see that one Cu will react with two AgNO3.

you need 0.0118 moles of Cu.
0.0235/2 = 0.0118

0.0118 x 62.55 = 0.738 grams of Cu

steam rated this answer Excellent or Above Average Answer

Question/Answer
malevon asked on 10/12/04 - Neutralization

How many grams of Mg(OH)2 will be needed to neutralize 25mL of stomach acid if stomach acid is 0.10M HCl?

I've gotten as far as balancing the equation, then I'm lost from there.

matrix answered on 10/12/04:

Hello Malevon
Where is your balanced equation???? We like to see you are doing some of the work.


Mg(OH)2 + 2 HCl ---> MgCl2 + 2 H2O

25.0 ml x 0.10 M = 2.5 mmoles of acid is needed to neutalize the Mg(OH)2.
You need 2 mmoles of acid for 1 mmole of Mg(OH)2. Therefore you will need 1.25 mmoles of Mg(OH)2.
1 mole of Mg(OH)2 is 58.33 grams
1 mmole of Mg(OH)2 is 58.33 milligrams
58.33x 1.25 = 72.9 milligrams.
This is rounded to 73 milligrams or 0.073 grams

malevon rated this answer Excellent or Above Average Answer

Question/Answer
steam asked on 10/07/04 -
Question 2

How many grams of chlorine gas must be reacted with excess sodium iodide if 10.0 g. of sodium chloride are needed?

matrix answered on 10/07/04:

Hello steam

2 NaI + Cl2 ----> 2 NaCl + I2

The equation tells you that it takes 1 mole of chlorine to form 2 moles of sodium chloride.
You want 10 g of NaCl. This is 10/58.44 = 0.17 moles of NaCl.

1/2 = X/0.17
X = 0.085 moles of Cl2 are needed.
Molecular weight of Cl2 is 70.91
0.085 x 70.91 = 6.03 rounded to 6.0 g of Cl2


Another way to do this.
The %Cl in NaCl is 35.45/58.44 x 100 = 60.7%
Thus in 10 g of NaCl you have 6.07g of Cl2.
[10g x 0.607] = 6.07g

steam rated this answer Excellent or Above Average Answer

Question/Answer
crimsonkristin asked on 09/27/04 - chem. formulas

Hi, what are the chemical formulas for these. I don't know how to figure them out, or how to start if any can give me some help and pointers on what to do I'd really appreciate it, thanks.

lead(II) nitrate solution reacts with sodium iodide solution to produce lead(II) iodide precipitate and sodium nitrate solution.

sulphuric acid reacts with sodium hydroxide to produce sodium sulphate solution and water.

barium chloride solution reacts with sodium chromate solution to produce barium chromate precipitate and sodium chloride solution.

When heated solid sodium hydrogen carbonate will decompose into solid sodium carbonate, carbon dioxide and water vapour.

matrix answered on 09/27/04:

lead(II) nitrate solution reacts with sodium iodide solution to produce lead(II) iodide precipitate and sodium nitrate solution.
Pb(NO3)2 + 2 NaI ----> PbI2 + 2 NaNO3


sulphuric acid reacts with sodium hydroxide to produce sodium sulphate solution and water.
H2SO4 + 2 NaOH ----> Na2SO4 + H2O

barium chloride solution reacts with sodium chromate solution to produce barium chromate precipitate and sodium chloride solution.
BaCl2 + Na2CrO4 ---> BaCrO4 + 2 NaCl

When heated solid sodium hydrogen carbonate will decompose into solid sodium carbonate, carbon dioxide and water vapour.
2 NaHCO3 ----> Na2CO3 + CO2 + H2O

Web Sites for equations:
http://www.cartage.org.lb/en/themes/Sciences/Chemistry/Inorganicchemistry/chemicslformula/Typesequation/Typesequation.htm

http://www.science.uwaterloo.ca/~cchieh/cact/c120/reaction.html



crimsonkristin rated this answer Excellent or Above Average Answer

Question/Answer
dapromise asked on 09/18/04 - Litmus Paper

What is litmus paper (made of) and how does it work? Thanks!

matrix answered on 09/19/04:

Litmus is a mixture of a number of phenolic materials extracted from lichens.
This material can exist in a number of structural forms that can interconvert by changing pH. Litmus paper is prepared by soaking paper in this extract and then drying it.
http://www.ibiblio.org/herbmed/eclectic/kings/roccella-tinc.html



I will give you a website that has the two forms of another common pH indicator, phenolphthalein.
http://www.sasked.gov.sk.ca/docs/chemistry/mission2mars/contents/chapter3/phenolphthalein.htm

dapromise rated this answer Excellent or Above Average Answer

Question/Answer
dapromise asked on 09/03/04 - Buffered Aspirin

The active ingredient in aspirin is acetylsalicylic acid. Why would doctors recommend buffered aspirin for some people, especially those who have a "sensitive" stomach?

matrix answered on 09/05/04:

Hello

Buffered aspirin is a combination of aspirin with some other compounds to reduce acidity. Some materials used as buffers in aspirin are magnesium oxide,magnesium carbonate and calcium carbonate.These materials react with some of the stomach acid (weak hydrochloric acid).This reduced acidicity retards the absorption of the aspirin by the stomach lining and allows it to pass into the intestines where it is absorpted and goes into the blood. Aspirin retards the production of compounds that stimulate blood clotting and thus causes capillaries in the stomach lining to leak. It is "easier" on the stomach if the aspirin is absorped by the intestine and NOT the stomach.
A better choice is to use enteric coated aspirin.This has a coating on the tablet that protects the aspirin until it passes through the stomach and into the intestines.

http://antoine.fsu.umd.edu/chem/senese/101/acidbase/faq/buffered-aspirin.shtml

dapromise rated this answer Excellent or Above Average Answer

Question/Answer
sanique asked on 04/12/04 - inorganic salts

How do you determine the waterof crystalization of an inorganic salt?

matrix answered on 04/12/04:

Hello

You need to very accurately weigh a sample of your compound in a preweighed suitable container (porcelain crucible). The crucible is then CAREFULLY heated to drive the water out.
After COOLING the crucible is weighed to find the weight of the water. The weight of the crucible is subtracted from the final weight of the crucible plus anhydrous sample to obtain the weight of the anhydrous sample.
Calculate the number of moles of water and of anhydrous sample. This gives your the ratio of compound to water of hydration.

Here are some web sites to read to help.
http://www.tvgreen.com/apchapt3/documents/bacllab.pdf
http://oldmanhonda.com/Chemistry/WebLabs/Hydrate/Hydrate.html

Question/Answer
adico asked on 04/01/04 - Acid base indicators

Dear Sir,
I was searching for examples of acid-base indiactors and I found them but I couldn't find their chemical structures , could you help me ?

matrix answered on 04/01/04:


See sites below.

http://antoine.frostburg.edu/chem/senese/101/features/water2wine.shtml

http://www.chemguide.co.uk/physical/acidbaseeqia/indicators.html

http://scidiv.bcc.ctc.edu/wv/elsp/Elsp.html

adico rated this answer Excellent or Above Average Answer

Question/Answer
adico asked on 03/31/04 - drugs

dear sir,
could you give me a name of drug contains one of these functional groups :
- aldehyde or ketons or amines

matrix answered on 03/31/04:

Quinidine contains both an aromatic and a tertiary alkyl amine group.

Amiodarone contains a ketone group.

See link below for structures.
http://lysine.pharm.utah.edu/netpharm/netpharm_00/figlist.htm

adico rated this answer Excellent or Above Average Answer

Question/Answer
adico asked on 03/29/04 - chemicals

Dear sir,
I want to ask about " Bromine water"
The composition
Uses
Sideeffect
Precaution

Thank you

matrix answered on 03/30/04:

Bromine water is made by dissolving bromine in water. It is usually about 3% (w/v) bromine.It can also be made by oxidizing a solution of bromide ion.
http://mattson.creighton.edu/BromineWater.html

It is used to test for the presence of unsaturation in organic compounds. The brownish (orange) bromine water is decolorized .
It is also used as a test for phenol.The addition of bromine water to a phenol gives a white precipitate and the orange color is discharged.

Elemental bromine is highly toxic, and touch or breathing more than trace amounts (10 ppm) can cause immediate health problems or death. Bromine is very irritating to both eyes and throat produces painful sores after making contact with skin. Improper handling of this element can be a serious health hazard requiring maximum safety precautions.

adico rated this answer Excellent or Above Average Answer

Question/Answer
adico asked on 03/24/04 - Chemicals

Dear sir,
I want to know about :
-Alum solution ( composition - uses )
-Millon's reagent ( composition - uses )

matrix answered on 03/24/04:

Hello Adico

I can help you with Millon's reagent.It is made by dissolving 1 part of mercury in 1 part of cold fuming nitric acid. This is diluted with twice its volume of water and decanting the clear solution after several hours.
A postive Millon's test is given by any compound containing a phenolic hydroxy group. Any protein containing tyrosine will give a positive test of
a pink to dark-red colour. The red colour is probably due to a mercury salt of nitrated tyrosine.


I need more detail about the alum solution.
Alum is an ambiguous term.There are a number of "alums". An alum is a double sulfate of a trivalent metal such as aluminum, chromium, or iron and a univalent ion such as potassium , sodium or ammonium . One of the more common alums is aluminum potassium sulfate, AlK(SO4)2�12H2O.
Alums are used medicinally as topical astringents.

adico rated this answer Excellent or Above Average Answer

Question/Answer
Sparkling_star asked on 03/07/04 - redox equation

what's the balanced equation for the redox titration between KMnO4 and Na2S2O3? pls help.. thank you.

matrix answered on 03/08/04:

Hello

Your reaction is below.

8 (MnO4)^- + H2O + 3 (S2O3)^2- ===> 8 MnO2 + 2 (OH)^- + 6 (SO4)^2-

Sparkling_star rated this answer Excellent or Above Average Answer

Question/Answer
dapromise asked on 02/24/04 - Vinegar

What does HC2H3O2, or vinegar, stand for? What does each letter represent? What elements does it contain? How does it combine and become HC2H3O2? If you know any websites, that would be good. Thank you and God bless!

matrix answered on 02/25/04:

Hello
I will try to answer the various parts of your question.Let me know if I missed something.

Vinegar is the name given to dilute solutions of acetic acid in water.It varies from 3 to 8% acetic acid in water.
The most common vinegar is 5% acetic acid.

I prefer that HC2H3O2 be written as C2H4O2 or CH3CO2H.This is the formula for acetic acid.It shows that each molecule of acetic acid contains 2 atoms of carbon (C),4 atoms of hydrogen (H) and 2 atoms of oxygen (O).
Another way of writing the formula is CH3CO2H.This way shows the arrangement of the atoms.One carbon has 3 hydrogens attached and this is attached to a second carbon with 2 attached oxygen atoms.

I will give you some web sites to read to help you understand this.
http://www.phatnav.com/wiki/wiki.phtml?title=Acetic_acid
http://en.wikipedia.org/wiki/Vinegar
http://scifun.chem.wisc.edu/chemweek/AceticAcid/AceticAcid.html

dapromise rated this answer Excellent or Above Average Answer

Question/Answer
STONY asked on 01/20/04 - NEED MORE SPECIFIC INFO...

I NEED A CLEARER DEFINITION OF UNSTABLE IN CONNECTION WITH THIS A/B POWDER. I'M ALWAYS LOOKING TO MAKE A BETTER BANG IN A HAND MADE FIRECRACKER; AND WE BRIEFLY DISCUSSED THE INSTABILITY FACTOR OF SUGAR AND PART A. MY QUESTION IS, DOES IT BECOME UNSTABLE IN AND OF ITSELF IF MIXED WITH SUGAR OR ONLY IN THE PRESENCE OF A HEAT SOURCE? I'M NOT A CHEMIST, I'M JUST A 55 YEAR OLD GUY WHO LIKES TO MAKE PARTY FAVORS FOR THE 4TH OF JULY AND NEW YEARS EVE.

matrix answered on 01/20/04:

Hello Tony

There are many causes of instability in the materials used in fireworks. I do NOT know and I have not seen the cause of instabity in this particular case of A/B powder when mixed with sugar but I will give one possilbility. Sucrose (common sugar), particular when finely divided, is slightly hygroscopic. Perhaps the resulting very slightly moist material is less stable than the dry material.


Unstable chemicals are those that are shock sensitive and explosive.Some shock sensitive chemicals are inherently capable of exploding mercury fulminate and metal azides such as lead azide. Shock sensitive chemicals can also degrade to an explosive state. The classic example of this is picric acid.When new it is stable but on standing over a number of years the material will explode when you open the bottle (crushing the crystals in the threads of the cap).

STONY rated this answer Excellent or Above Average Answer

Question/Answer
STONY asked on 01/07/04 - ANOTHER DUMB {?} BECAUSE I'M NOT A CHEMIST.

I READ IN A WARNING SOMEWHERTHAT YOU SHOULD NOT MIX FLASH POWDER WITH ORGANICS BECAUSE IT WOULD BECOME UNSTABLE. DOWN HERE IN FLA. THE HUMIDITY IS SO HIGH THAT SOMETIMES CLUMPING IS PROBLEM. I KNOW IN CONNFECTIONERS SUGAR THEY USE A SMALL {MAYBE 3%} AMOUNT OF CORNSTARCH TO PREVENT CLUMPING, COULD YOU DO THE SAME THING IN FLASHPOWDER?

matrix answered on 01/07/04:

Hello Tony
I do NOT know the answer to your question but keep in mind that starch is a polysaccharide (a polymer of sugar). Mixtures of potassium chlorate and sucrose (regular sugar) are very dangerous (unstable and too sensitive) and mixtures of potassium perchlorate and sugar are explosive.
I do NOT know if a polymer of sugar will react the same as sugar. My guess is that it will not be as reactive as sugar.

I do NOT know if it will prevent clumping of flash powder.Perhaps you can run experiments. Make two batchs of flash powder,one with and one without starch. Check to see if the starch helps.

STONY rated this answer Excellent or Above Average Answer

Question/Answer
Quimbly asked on 01/05/04 - Compressibility of Water

Hi,
I've been trying to understand the compressibility of liquids. I can't seem to find anything definitive on the Internet on the subject. All I get is that liquids are practically incompressibile. However, I AM interested in how much they do compress, regardless of how small it is, or how much pressure is needed.

One figure I could was as follows: 0.46 GPa-1

Does this mean that water under 1 GPa (GigaPascal) of pressure will compress by a volumetric factor of 0.46??

If you think it is correct, can you verify this? Do you have a link to some site where this is docu*mented?

Also, I wouldn't think that compression would be a linear function of pressure. Is it? If not, does anyone have a graph that shows water's (or other liquid's) volume as a function of pressure?

Thanks!

matrix answered on 01/05/04:

Hello Quimbly

Compressibility is defined as the fractional change in
volume per unit increase in pressure. In your
example of water, for each atmosphere increase in pressure,
it's volume will decrease by 46.4 parts per million.

Here are some links that briefly discuss this.
http://hyperphysics.phy-astr.gsu.edu/hbase/tables/compress.html

http://www.ps.uci.edu/physics/news5/yu5.html

Bulk Modulus of Elasticity
http://www.shirazu.ac.ir/schools/eng/chem-dept/courses/fluid/chap1.htm

Question/Answer
STONY asked on 12/23/03 - THE A/B POWDER OF TEN YRS AGO

MUST HAVE BEEN MADE WITH THAT HCLO CONFIGURATION.
I TRIED THAT HCLO4 STUFF AND WITH SOME ATOMIZED BLACK GERMAN ALUMINUM AND THE "BANG" WAS SO LOUD IT ALMOST TOOK YOUR BREATH AWAY. AND I DID NOT USE A PILE OF IT EITHER. THANK YOU FOR DESCRIBING THE 4 TYPES.

matrix answered on 12/29/03:

Hello Stony

Of the four potassium oxychloride salts only
the chlorate(KClO3) and perchlorate(KClO4)are commonly
used in fireworks.The perchlorate is considered the safest.


http://www.geocities.com/CapeCanaveral/Lab/1121/chemicals.html
The material below comes from the link above.

Potassium chlorate(KClO3)
Originally used very commonly in pyrotechnics, potassium chlorate has
gradually been phased out due to its sensitivity, in favor of potassium
perchlorate. Mixtures containing potassium chlorate and ammonium salts,
phosphorus or anything acidic are particularly dangerous. For this reason
mixtures containing potassium chlorate and sulphur are to be avoided,
as sulphur (especially the common "flowers" of sulphur) may contain
residual amounts of acid that can sensitize the mixture. In general,
potassium chlorate should be avoided unless absolutely necessary.
Chlorates have probably caused more accidents in the industry than all
other classes of oxidisers together. The reason lies in their sensitivity
to acids and their low decomposition temperature. When mixed with an
easily ignitable fuel, such as sugar or sulfur, chlorates will ignite
from a fingernail striking a wire screen. Moreover, sulfur is often
acidic, a fact that has lead to spontaneous ignition of sulfur-chlorate
compositions. If you intend to use chlorates, pay extra attention to safety.

Potassium perchlorate(KClO4)
More expensive than potassium chlorate, but a better oxidising agent
and far safer. In almost all mixtures that previously required the
chlorate, safety factors have led to its replacement with potassium
perchlorate. It should be used in place of the chlorate wherever possible.
Perchlorates are safer to handle than chlorates; they are more stable when
exposed to heat or shock.





STONY rated this answer Excellent or Above Average Answer

Question/Answer
STONY asked on 10/20/03 - WHAT DOES PERCHLORATE MEAN?

AS IN POTASSIUM PERCHLORATE...

matrix answered on 10/20/03:

Hello STONY
In naming oxyacids and their salts, the use of per means it contains the highest amount of oxygen.This is because there are a number of different oxyacids where the difference is the number of oxygens in the molecule.
For example in the following
KClO4, potassium perchlorate (higher oxygen content)
KClO3, potassium chlorate (normal oxygen content)
KClO2, potassium chlorite (lower oxygen content)
KClO, potassium hypochlorite (even lower oxygen content)
This is one of the things you will need to memorize.

I will give some sites with this nomenclature.
http://www.chem.umn.edu/class/1051/old/spring/namerule.pdf.

http://www.ucdsb.on.ca/tiss/stretton/chem1/atomic7.html

http://basd.k12.wi.us/~kreutz/A5-oxyAcidNaming.ht

STONY rated this answer Excellent or Above Average Answer

Question/Answer
yo_lam asked on 10/20/03 - Orbitals

I don't quite understand this...

Is it true that Group VI element, oxygen, has six valence electrons and two core, i.e. 1s, electrons?

Secondly, in a neutral carbon atom, are the 2s and 2p orbitals all degenerate?

matrix answered on 10/20/03:

Hello

Valence electrons are the electrons in the last shell or energy level of an atom.In the case of oxygen there are 6 valence electrons.

Carbon in its ground state has an electronic structure of 1s2 2s2 2p2.Two electrons occupy the 1s orbital, two occupy the 2s
orbital, and two occupy 2p orbitals. Since there are three 2p orbitals with the same energy, the mutually repulsive electrons will occupy separate 2p orbitals. The 2s and 2p wavefunctions are normally hybridised to form four sp3 degenerate orbitals.
http://hmchemdemo.clt.binghamton.edu/zumdahl/docs/chemistry/07atomstructure/library/0711.ht

Question/Answer
itz_jacki asked on 10/20/03 - Number of Isomers

How exactly can you determine the number of isomers of a given chemical formula for a molecule say, C6H14 Hexane? Am I supposed to draw them all out and see or is there a systematic way of doing things?

matrix answered on 10/20/03:

Hello

There is no easy way to determine the number of isomers of possible for a given formula.
Drawing the possible structures and eliminating those that are duplicates is the only way to do this.
The number of possible isomers increases rapidly as the number of carbons increases.
For example;
Pentane, C5H12,has 3 isomers
Hexane, C6H14,has 5 isomers
Heptane, C7H16,has 9 isomers
Octane, C8H18,has 18 isomers
Nonane, C9H20,has 35 isomers

In the case of C6H14 they are;
CH3CH2CH2CH2CH2CH3

(CH3)2CHCH2CH2CH3

(CH3CH2)2CHCH3

(CH3)2CHCH(CH3)2

(CH3)3CCH2CH3

Please rate all answers you receive to ALL of your question

itz_jacki rated this answer Excellent or Above Average Answer

Question/Answer
itz_jacki asked on 10/12/03 - pH levels

I don't understand why...

In water, it is not possilbe to have an acidic solution with a pH < 0 or a basic solution with a pH > 15

Thanks

matrix answered on 10/13/03:

IN WATER SYSTEMS.
Acids donate a proton to water to form the hydronium ion (H3O+) and bases accept a proton from water to form the hydroxide ion (OH-).
Strong acids are those that COMPLETELY DISSOCIATE into H3O+ and A- and strong bases
that COMPLETELY ASSOCIATE with protons from water to form HB and OH-.
THE STRONGEST ACID THAT CAN EXIST IN WATER IS H3O+ AND THE STRONGEST BASE THAT CAN EXIST IN WATER IS OH-.

The pH range is thus set because of the following relationship.
Since K = [H+][OH-} = 14
pH + pOH = 14

The following sites may help.
http://bcn.boulder.co.us/basin/data/COBWQ/info/pH.html
http://www.narrabay.com/Documents/PDFs/phTheoryInsert.pdf



If you want to see the different acid strengths of strong acids, then you must study these acids in a solvent other than H2O. There are acids much stronger than sulfuric acid. Some are as much as 10^18 times stronger that 100% sulphuric acid.

Question/Answer
itz_jacki asked on 10/01/03 - Reaction

I mixed 2-naphthol and NaOH together and aparently, I see some cloudy mixture, which I think is precipitate. Does that mean a reaction occured?

matrix answered on 10/01/03:

Sodium hydroxide reacts with 2-naphthol to forms it's sodium salt (which is H2O soluble).
If you add acid to this mixture you will reform 2-naphthol.

You did not describe your procedure. Did you try to dissolve the naphthol in water and add sodium hydroxide solution? 2-Naphthol has LOW solubility in H2O. Perhaps all of the naphthol had not dissolved . Adding base to this will INCREASE the solubilty of the naphthol (it forms the salt).

Question/Answer
girlie123 asked on 09/28/03 - Atomic mass

Thank you for your previous answer. I have one more question, hopefully you can help me answer it.

The atomic mass of element X is 33.42 amu. A 27.22g sample of X combines with 84.10g of another element Y to form a compound XY. Calculate the atomic mass of Y.

Thank you for any suggestions. :)

matrix answered on 09/28/03:

X + Y = XY
27.22/33.42 = 0.8145 moles of X reacted.
It reacted with 0.8145 moles of Y.
Thus 84.10 g is 0.8145 moles of Y.
84.10/Y = 0.8145
Y = 103.3...This is atomic mass of Y

girlie123 rated this answer Excellent or Above Average Answer

Question/Answer
girlie123 asked on 09/27/03 - Stoichiometry

The aluminum sulfate hydrate [Al2(SO4)3 x XH20] constains 8.20 percent of Al by mass. Calculate X, that is the number of water molecules associated with each Al2(SO4)3 unit.

Please any suggestions will help.

Thank you.

matrix answered on 09/27/03:


Atomic weight of Al is 26.98
In your compound you have 2 Al atoms.
26.98 x 2 = 53.96
This is also 8.20% of the mass of your compound.
8.20% of X = 53.96
X = 658.05 is molecular mass of your compound including the water.

Al2(SO4)3 has mass of 342.15

658.05 - 342.15 = 315.9....This is mass of all of the H2O.

315.9/18 = 17.55...round to 18

Your compound is Al2(SO4)3*18H2O

girlie123 rated this answer Excellent or Above Average Answer

Question/Answer
itz_jacki asked on 09/19/03 - Phenol

Hi,
I just want to know, is p-toluidine, benzoic acid, and/or 2-naphthol considered a phenol?

Thanks

matrix answered on 09/19/03:


Hello

Toluidine is an aromatic amine.
Benzoic acid is an aromatic carboxylic acid.

Only 2-Naphthol is a phenol.
It's structure is at the bottom of the following link.
http://www.rod.beavon.clara.net/2-naphth.htm

Question/Answer
linapenunuri7 asked on 08/28/03 - Calculating joules

How many joules does it take to raise the temperature of 2.00L of water from 22.0 degree celcius to 40.0 degree celcius? Density of water is 1.00 g/mL.

what formula am i suppose to use? i thought:
Heat= specific heat of water x mass of water in calorimeter x change in water temp

How do i convert 2.0 L of water to be able to add it in the fomula?

matrix answered on 08/28/03:

Hello
Volume x density = mass
2000 ml x 1.00 g/ml = 2000 g of H2O

Temp change is 40.0 - 22.0 = 18.0�C
Specific heat of H2O is 4.184 J/g �C

Heat = 4.184 x 2000 x 18.0 = 150,624 J
This should be rounded to 3 places --> 151,000 J

Question/Answer
slimorish2003 asked on 07/25/03 - dishwashing liquid + environment

what chemicals does dishwashing liquid contain that are harmful to the environment and why are they destructive to the environment?

matrix answered on 07/28/03:

Hello

I cannot give you a good answer to your question because I do not know all the chemicals in dishwashing liquids.
I know some of them contain diethanolamine,cocamide and lauramide which there is some evidence of a cause of liver
and kidney cancer.

The most harmful are the automatic dishwashing powders that contains
harsh detergents with high concentrations of phosphates.
Released into lakes and streams, phosphates kill fish and other aquatic life.

Phosphorus is a nutrient for plant growth � a nutrient
that in excessive amounts can cause algae growth.
spots. Because one pound of phosphorus can grow up to 700
pounds of algae, it can cause tremendous damage to
lakes and streams.Increased decomposition of algae can deplete the oxygen needed by
fish and other aquatic organisms to survive.

slimorish2003 rated this answer Excellent or Above Average Answer

Question/Answer
~cherub~ asked on 07/22/03 - Steel and carbon dating

I can't get the "follow-up" to work...


I am in a discussion on "steel" and its dating on one of the Christianity forums. My question is this... how do I know that a steel artifact that is claimed to be dated at 1200 BCE is actually dated correctly. How big a sampling does it take to get an accurate dating based upon the carbon content. And doesn't the carbon content deteriorate over time, you know corrosion and rust away?

~cherub~

matrix answered on 07/23/03:

I do not do C-14 dating so I cannot give a required sample weight BUT I do know it will be more than a gram.The problem with carbon dating steel samples is the very low amount of carbon that is in steel ,usually less than 1%.
I checked with some C-14 labs for their sample size requirements and I will list some links to their web sites.


You will also see mention of a new technique for carbon dating called accelerator mass spectrometry (AMS).This procedure requires much less sample but it will probably be multiple milligram amounts for steel samples.They need around one mg "of carbon" (not steel) for their work.


http://id-archserve.ucsb.edu/Anth3/Courseware/Chronology/08_Radiocarbon_Dating.html
http://www.geochronlabs.com/14c.html


Also corrosion and rust does NOT affect the ratio of carbon isotopes in a sample.Radioactive decay is a nuclear process and is NOT affected by chemical reactions.

Jim.McGinness rated this answer Excellent or Above Average Answer
~cherub~ rated this answer Excellent or Above Average Answer

Question/Answer
~cherub~ asked on 07/22/03 - carbon dating....

Okay,

I forgot which of you stated this....

"Early steel makers used charcoal that had been made from the trees of that period."


If I am looking for the earliest dates of steel being used, then these artifacts could be carbon dated with accuracy because trees were used to obtain the carbon?


Is there a timeframe there? Or, are there internet sites that explain the oldest of steel's origins?


~cherub~

matrix answered on 07/22/03:

Hello cherub

Steel was first made approximately 3000 years ago.The first recorded steel making was by the Chalybes in Pontus in Asia Minor around the twelfth century B.C.
Steel from this time period can be carbon dated but the accuracy of this dating may not be as high as that of items having a higher carbon content. Steel contains less than 1% carbon so a fairly large sample size is needed. Also the steel sample needs to be dissolved in acid and the carbon collected in some form that can be oxidized.The percent recovery of this process is around 80-90%.

http://www.bbc.co.uk/dna/ww2/A647219

Jim.McGinness rated this answer Excellent or Above Average Answer
~cherub~ rated this answer Excellent or Above Average Answer

Question/Answer
~cherub~ asked on 07/20/03 - Concerning DNA

This might seem like a "dumb" question, but does steel have a DNA?

Can steel be identified in time by its molecular structure?

Thanks~

~cherub~

matrix answered on 07/21/03:

Hello cherub
No,steel does not contain DNA.
DNA is the genetic material producted by
living organisms.

Steel is iron containing a small amount of
carbon. It is a mixture and does NOT has a
molecular structure. The various kinds of
steels contain varying amounts of carbon
along with a number of other elements.These
various kinds of steels can also be given
different properties depending on the various
heating/cooling treatment they receive.The
hardening of steels depends on altering the
crystal structure of the steel by heating it
and then "freezing" the steel in the new crystalline
configuration by cooling it so quickly that
it can't go back to the crystal structure
that it used to have before hardening.

If you want to compare two samples of steel
to see if they are "related" then you would
could analyze them for chemical composition.
Another common way to compare steel samples
is to polish the samples and compare them under a
microscope ( 50 to 100 power).The different
crystalline structures can be viewed to see
if the samples are similar.

~cherub~ rated this answer Excellent or Above Average Answer

Question/Answer
cyberbatt asked on 06/28/03 - hybirdization

I had just learnt what hybirdization is but do not understand quite some parts. Please explain to me.
What hybirdization is? How can a carbon atom get excited and form sp3 species? You may like to explain by using ethane or ethene as an example.

matrix answered on 07/01/03:

Hello cyberbatt

The phenomenon of mixing up of different
orbitals of same energy level of an atom to
produce EQUAL NUMBER OF HYBRID-ORBITALS OF
SAME ENERGY AND IDENTICAL PROPERTIES is known
as hybridization.
The four orbitals around carbon hybridize to
form four equivalent orbitals having 75%
p-character and 25% s-character. The driving
force behind hybridization is a LOWERING OF
ELECTRONIC REPULSION. By placing the four
orbitals in tetrahedral geometry provides
the MAXIMUM SEPARATION BETWEEN THE ELECTRON
PAIRS AND MINIMIZES ELECTRONIC REPULSION.
The hybridization of one s- and three p-orbitals
is termed an sp3 hybrid and an sp3 center
should always be considered to have approximate
tetrahedral geometry,with the overriding factor
being the driving force for the molecule to
assume the lowest energy geometry, which is
readily accessible. Remember that the p orbitals
normally lie along x,y,z axes.Thus if the unhybridized
orbitals formed bonded with s orbitals the resulting
bond angles would be 90�.By forming the hybridized
orbitals the bond angle can be spread to ~109�.This
means the electronic repulsion is lowered.

The hybridization of one s orbital and two p
orbitals on a central atom gives rise to three sp2
orbitals. Hybridization as sp2 gives three orbitals,
which are planar and 120� apart. Thus for sp2 hybridization,
the 2s and only two of the 2p orbitals combine to make three
major lobes in a trigonal planar geometry. The third
2p orbital, orthogonal to the three sp2 lobes, does
not take part in hybridization. It is available to take part in
bonding as a regular 2p orbital. Thus sp2 atoms
usually take part in double bonds.

It is easier to understand this with drawings of orbitals.
I will give you some sites with drawings to help you.

http://chipo.chem.uic.edu/web1/ocol/SB/1-3.htm
http://www.towson.edu/~ladon/carbon.html
http://www.thinkquest.org/library/lib/site_sum_outside.html?tname=C004970&url=C004970/atoms/shapes.htm
http://www-personal.une.edu.au/~sglover/CHEM110TopicAB/sld013.htm

cyberbatt rated this answer Excellent or Above Average Answer
rainbow1000 rated this answer Excellent or Above Average Answer

Question/Answer
Navspert asked on 06/11/03 - Blow up!

Hello Matrix.

Please tell me how dynamite works. How is the reaction initiated?

Ps. If dynamite does decompose, then can it be used in a place of little or no oxygen like outer space? (Of course, gasoline can not combust in outer space).

Thank you!

matrix answered on 06/11/03:

Hello

Dynamite is nitroglycerine that has been desensitized
by mixing it with an inert material. Nobel found that
mixing nitroglycerine with kieselguhr (a mineral
containing a large amount of silica) would turn the
liquid into a paste which could be shaped into rods
of a size and form suitable for insertion into
drilling holes - and which only exploded when they
were detonated. In 1867 he patented this material
using the name dynamite (from the Greek word meaning 'power').
This was the first safe high explosive and it's discovery made
Nobel very rich. It later established the Nobel prizes
with the riches he had made from dynamite.
The dynamite of today is not the same formula as Nobels.
In the United States, the standard dynamite is composed of nitroglycerin,
ammonium nitrate and sodium nitrate, wood pulp, and a little bit
of calcium carbonate which neutralizes acids that may develop during storage.

Dynamite does NOT need an external oxygen source.
It is the VERY QUICK conversion of the nitroglycerin into very large
amounts of hot gases that give the explosive it's get power.

4 C3H5N3O9 ---> 10 H2O (g)+ 12 CO2 (g) + 6 N2 (g) + O2 (g)

Thus from 4 moles of nitroglycerine (908 grams,568 ml) you form
29 moles of HOT gases with a volume of more than 2550 liters(assume
gas temp of 800�C).This is a 4400 times increase in volume in a very short
time.

Navspert rated this answer Excellent or Above Average Answer

Question/Answer
Navspert asked on 05/30/03 - Heat or ""

Please give me a general algorithm to find enthalpy of a solution or substance,

I appreciate it!

matrix answered on 06/04/03:

Hello
Sorry to have taken so long to answer your question but I have been away on vacation.

I was going to decline your question but thought this may help. I prefer not to answer questions dealing with thermodynamics because I am not comfortable with my answers. However, please check the following site to see if this is any help.
http://puccini.che.pitt.edu/~karlj/Classes/CHE1007/l13/

Navspert rated this answer Excellent or Above Average Answer

Question/Answer
Squall asked on 05/11/03 - Bromophenol Blue

I think the lab may have played a pratical joke on me because I'm whizzing blue urine. I'm pretty sure its because they may have spiked my drink with bromophenol blue. Should I be worried? No hazard symbols on the vial so is it safe? Any ideas for revenge?

matrix answered on 05/11/03:

Hello

I have not heard of using bromophenol blue to do this. I am assuming they gave you a VERY SMALL amount. Large oral doses may cause irritation to the gastrointestinal tract.I will give you the MSDS for this.

http://physchem.ox.ac.uk/MSDS/BR/bromophenol_blue.html


The material usually used in this prank is methylene blue. This is a component in several medications (Trac Tabs, Urised, Uroblue) used to reduce symptoms of cystitis. It is not toxic.

Squall rated this answer Excellent or Above Average Answer

Question/Answer
Navspert asked on 04/22/03 - Fire Heat, Where does it come from?

Hello Mr. Matrix
In this reaction 2H2+O2-->2H2O, lots of heat is evolved and my question is: Where does this heat come from? This simple question has been plaguing me since forever!! I'd prefer a detailed explanation on the phenomenon of fire heat.

PS Is your answer true for all combustion reactions?

Thank you, very very much!!

matrix answered on 05/02/03:

Hello

Sorry to have taken so long to answer your question
but I have been away on vacation and I am slowly
catching up on all that I missed while I was
away.

The simple answer to your question question
is that the energy content of the products
(in this case H2O) is less than that of the
starting materials. All substances a certain
amount of energy.Substances that react form new
substances with a lower energy content(exothermic
reactions).Both hydrogen and oxygen have a relatively
high energy content (they are reactive materials).They
will react to form water,a substance that is much less
reactive.The difference in energy content between the
mixture of hydrogen and oxygen and that of water(steam)
is the energy given off.
Also remember that as steam condenses to form liquid water
an additional amount of energy is released (540 calories/gram,
the heat of vaporization).

This is true for combustion reactions.The starting materials
react to form water,CO2,etc which have a lower energy content.

http://www.chem.uidaho.edu/~honors/rate3.html

Navspert rated this answer Excellent or Above Average Answer

Question/Answer
jenny96 asked on 04/23/03 - Nitrate

Potassium

Equal volumes of 0.120 mol/L potassium nitrate and 0.160 mol/L iron (iii) nitrate are mixed together. What is the concentration of nitrate ions in the mixture?

matrix answered on 04/29/03:

Hello Jenny

HAVE YOU BEEN READING MY ANSWERS ??? Since you have not rated the answers I will think you are not reading my answers.



To help to this problem I will assume you have one liter of each solution.You could use any volume as long as you divide the number of moles by the number of liters.

1 liter x 0.120M KNO3 contains 0.120 moles of NO3- ion.

1 liter of 0.160M Fe(NO3)3 contains (3 x 0.160) 0.480 moles of NO3- ion.

You have a total of (0.120 + 0.480) 0.600 moles of NO3- ion. This is in a total volume of 2 liters.

0.600 moles/2 liters = 0.300 M is concentration of NO3- ions.

jenny96 rated this answer Excellent or Above Average Answer

Question/Answer
jenny96 asked on 04/23/03 - Aluminium

Aluminium

An excess of aluminum foil is added to a certain volume of 0.675 mol/L aqueous copper (ii) sulfate solution. The mass of solid copper that precipitates is measured and found to be 4.88 g. What was the volume of the copper (ii) sulfate solution?

matrix answered on 04/29/03:

2 Al + 3 CuSO4 ---> Al2(SO4)3 + 3 Cu

4.88/63.55 = 0.0768 moles of Cu formed
This is 76.7 mmoles of Cu

(0.675M)(X ml) = 76.7 mmoles
x = 76.7/0.675 = 113.6 ---> 114 ml of CuSO4 solution.


NOTE: It is copper(II) sulfate not copper (ii) sulfate.

jenny96 rated this answer Excellent or Above Average Answer

Question/Answer
jenny96 asked on 04/23/03 - Hydrogen

Hydrogen

To generate hydrogen gas, a student adds 5.77g of mossy zinc to 80.1 mL of 4.00 mol/L hydrochloric acid in an Erlenmeyer flask. When the reaction is over, what is the concentration of aqueous zinc chloride in the flask?

matrix answered on 04/29/03:

Hello Jenny
Sorry to be so long in answering your question but I have been away on vacation.

5.77/65.41 = 0.0882 moles of Zn

80.1ml x 4.00M HCl = 320.4 ----> 320 mmoles of HCl = 0.320 moles of HCL

2 HCl + Zn ---> ZnCl2 + H2

From equation you can see you need 2 moles of HCl for each mole of Zn. Thus you need to have
2 x 0.0882 = 0.176 moles of HCl. Since you have 0.320 moles the HCl is in excess and the zinc is the limiting reagent.
You will form 0.0882 moles of ZnCl2. This will be in 80.1 ml or 0.0801 liters.
0.0882/0.0801 = 1.01 molar ZnCl2

CHECK ALL MATH !!!!

jenny96 rated this answer Excellent or Above Average Answer

Question/Answer
jenny96 asked on 04/18/03 - aqueous

last question
An aqueous solution of a certain salt contains chloride ions. A sample of this solution was made by dissolving 17.59 g of the salt in a 1 L volumetric flask. Then 25.00 mL of the solution was treated with excess silver nitrate. The precipitate, AgCl(s), was filtered and dried. If the mass of the dry precipitate was 47.35 g, what was the mass percent of chloride ions in the solution?

matrix answered on 04/23/03:

Hello Jenny
I believe you have an error in your question.

AgNO3 + Cl- ---> AgCl
Molar mass of AgCl is 143.32
Molar mass of Cl- is 35.45.

The mass of Cl- in the 47.35 grams of dried AgCl is calculated by multiplying the dry mass by the ratio of Cl-/AgCl
47.35 x (35.34/143.32) = 11.71 grams Cl- in sample of AgCl.

THE PROBLEM I AM HAVING WITH YOUR QUESTION IS THAT THE MASS OF Cl- IN THE DRIED AgCl IS TOO HIGH FOR THE AMOUNT OF SOLUTION (25ml) YOU ARE USING.
17.59 grams in 1000ml
YOU STATE THAT ONLY 25ml OF THE SAMPLE IS BEING USED.
25.00ml is 25.00/1000 = 0.0250
17.59 x 0.0250 = 0.4398g is mass of salt that was reacted with silver nitrate.

USING YOUR NUMBERS 0.4398 grams OF SALT GAVE 47.35 grams OF AgCl OF WHICH 11.71 grams IS Cl-.

THIS IS NOT POSSIBLE !!!!!

Unless you have more details this question is ended.

jenny96 rated this answer Excellent or Above Average Answer

Question/Answer
jenny96 asked on 04/18/03 - aqueous

last question
An aqueous solution of a certain salt contains chloride ions. A sample of this solution was made by dissolving 17.59 g of the salt in a 1 L volumetric flask. Then 25.00 mL of the solution was treated with excess silver nitrate. The precipitate, AgCl(s), was filtered and dried. If the mass of the dry precipitate was 47.35 g, what was the mass percent of chloride ions in the solution?

matrix answered on 04/23/03:

Hello Jenny
I believe you have an error in your question.

AgNO3 + Cl- ---> AgCl
Molar mass of AgCl is 143.32
Molar mass of Cl- is 35.45.

The mass of Cl- in the 47.35 grams of dried AgCl is calculated by multiplying the dry mass by the ratio of Cl-/AgCl
47.35 x (35.34/143.32) = 11.71 grams Cl- in sample of AgCl.

THE PROBLEM I AM HAVING WITH YOUR QUESTION IS THAT THE MASS OF Cl- IN THE DRIED AgCl IS TOO HIGH FOR THE AMOUNT OF SOLUTION (25ml) YOU ARE USING.
17.59 grams in 1000ml
YOU STATE THAT ONLY 25ml OF THE SAMPLE IS BEING USED.
25.00ml is 25.00/1000 = 0.0250
17.59 x 0.0250 = 0.4398g is mass of salt that was reacted with silver nitrate.

USING YOUR NUMBERS 0.4398 grams OF SALT GAVE 47.35 grams OF AgCl OF WHICH 11.71 grams IS Cl-.

THIS IS NOT POSSIBLE !!!!!

Unless you have more details this question is ended.

jenny96 rated this answer Excellent or Above Average Answer

Question/Answer
jenny96 asked on 04/23/03 - Stomach1

Hydrocloric acid

Your stomach secretes hydrochloric acid to help you digest the food you have eaten. If too much HCl is secreted, however, you may need to take an antacid to neutralize the exces. One antacid product contains the compound magnesium hydroxide, Mg(OH)2.

a)Predict the reaction that takes place when magnesium hydroxide reacts with hydrochloric acid. (Hint: This is a double displacement reaction.)

b) Imagine that you are a chemical analyst testing the effectiveness of antacids. If 0.10 mol/L HCl serves as your model for stomach acid, how many litres will react with an antacid that contains 0.10 g of magnesium hydroxide?

matrix answered on 04/23/03:

Hello Jenny

(a) Mg(OH)2 + 2 HCl ---> MgCl2 + 2 H2O

(b) 0.10 grams /58.32 = 0.001715 moles of Mg(OH)2

(0.10 molar HCl)( X liters) = 0.001715 moles

X liters = 0.001715/0.10 = 0.01715 liters = 17.15 ml of 0.10M HCL

jenny96 rated this answer Excellent or Above Average Answer

Question/Answer
jenny96 asked on 04/23/03 - Metallic Iron

Acid

Copper can be recovered from scrap metal by adding sulfuric acid. Soluble copper sulfate is formed. The copper sulfate then reacts with metallic iron in a single displacement reaction. To simulate this reaction, a student places 1.942 g of iron wool in a beaker that contains 136.3 mL of 0.0750 mol/L aqueous copper (ii) sulfate. What mass of copper is formed?

matrix answered on 04/23/03:

Hello Jenny

Fe + CuSO4 ----> FeSO4 + Cu

1.942/55.85 = 0.03477 moles of iron

136.3 ml x 0.0750 = 10.22 mmoles = 0.01022 moles of Cu in solution.

You have excess Fe. All of the Fe will NOT react.

All of the Cu will be displaced from solution.

0.01022 moles of Cu will be displaced.
0.01022 x 63.55 = 0.6495 grams of Cu

CHECK ALL MATH !!!!

jenny96 rated this answer Excellent or Above Average Answer

Question/Answer
jenny96 asked on 04/18/03 - Precipitate question

Fourth question

25.0 mL of 0.400 mol/L Pb(NO3)2(aq) is mixed with 300 mL of 0.200 mol/L KI(aq). What is the maximum mass of precipitate that can form?

matrix answered on 04/18/03:

Hello Jenny

Pb(NO3)2 + 2 KI ---> PbI2 + 2 KNO3

25.0ml x 0.400M = 10.0 mmoles of Pb(NO3)2
300ml x 0.200M = 60.0 mmoles of KI

From equation one mole of Pb(NO3)2 requires two moles of KI. Since you have 10.0 mmoles of Pb(NO3)2 you will need 20 mmoles of KI.
Since you have 60.0 mmoles of KI, this reagent is in excess.
All of the Pb(NO3)2 will react.
This will form 10.0 mmoles of PbI2.
The mass of a mmole of PbI2 is 461 mg.
461 x 10.0 = 4610 mg or 4.61 grams

jenny96 rated this answer Excellent or Above Average Answer

Question/Answer
jenny96 asked on 04/18/03 - ammonium

can you help

Ammonium phosphate can be used as a fertilizer. 6.0 g of ammonium phosphate is dissolved in sufficient water to produce 300 mL of solution. What are the concentrations (in mol/L) of the ammonium ions and phosphate ions present?

matrix answered on 04/18/03:

Hello Jenny

Molecular mass of (NH4)3PO4 is 149.096

6.0/149.096 = 0.040 moles
molarity is moles per liter.
0.040/0.30 = 0.13 molar...This is the molarity of (NH4)3PO4 in the solution.
The molarity of phosphate in the solution is 0.13M
The molarity of the ammonium ion is 3 x 0.13 = 0.39M

jenny96 rated this answer Excellent or Above Average Answer

Question/Answer
jenny96 asked on 04/18/03 - Active ingredient

Third question
The active ingredient in some rat poisons is thallium (I) sulfate, Tl2SO4. A chemist takes a 500 mg sample of thallium (I) sulfate and adds potassium iodide, to precipitate yellow thallium (I) iodide. When the precipitate is dried, its mass is 200 mg. What is the mass percent of Tl2SO4 in the rat poison?

matrix answered on 04/18/03:

Hello Jenny

Tl2SO4 + 2 KI ---> 2 TlI + K2SO4

TlI...331.27...molecular wt.
Tl2SO4...504.8....molecular wt

0.200/331.27 = 0.000604 moles of TlI

One mole of Tl2SO4 gives 2 moles of TlI
Xmoles of Tl2SO4 gave 0.000604 moles of TlI
1/2 = X/0.000604
x = 0.000302 moles of Tl2SO4 in sample.

0.000302 x 504.8 = 0.152g = 152mg of Tl2SO4 in the sample.
(152/500) x 100 = 30.4% Tl2SO4 in sample

Check all math and m.wt.

jenny96 rated this answer Excellent or Above Average Answer

Question/Answer
jenny96 asked on 04/18/03 - Question1

1. Food manufacturers sometimes add calcium acetate to puddings and sweet sauces as a thickening agent. What volume of 0.500 mol/L calcium acetate, Ca(CH3COO)2(aq), contains 0.300 mol of acetate ions?

matrix answered on 04/18/03:

Hello Jenny

You have a solution of 0.500M Ca(OAc)2.This means the solution
contains 0.500 moles of Ca(OAc)2 in a liter of solution.
Since each molecule of Ca(OAc)2 contains two acetates ,the solution
is actually 1.00M in acetate ion.That means a liter of solution
contains 1.00 moles of acetate.
Since you need 0.300 moles of acetate you will need
300 ml of solution.

(0.300/1.00) x 1000ml = 300ml

jenny96 rated this answer Excellent or Above Average Answer

Question/Answer
Navspert asked on 04/06/03 - What's ppm?

Hello,

I was reading my chemistry text and found this abbreviation: ppm. What does this mean? Please give a thorough explanation.

Thank you very much!

matrix answered on 04/06/03:

ppm is abbreviation for parts per million.

Parts per million is used when you are working with a small amount of one substance mixed with a larger amount of a second substance.
PPM is defined as the mass of the component in solution divided by the total mass of the solution multiplied by 10^6 (one million):
1 ppm = 1 milligram per liter (mg/L)
1 ppm = 3.8 milligram per gallon (mg/gal)

Also, one percent = 10,000 ppm.

Navspert rated this answer Excellent or Above Average Answer

Question/Answer
cyberbatt asked on 03/19/03 - bonding

I was taught that when light shine on a metal, the delocalised electrons are excited to higher energy levels. The return of the electrons to the original energy level releases light energy, which is perceived as the shine in metals but why iodine is also shiny in appearance although there are no delocalised electrons?

matrix answered on 03/21/03:

Hello cyberbatt

The absorption of light by the free electrons
followed by it's reradiation is responible for the
shiny appearance of metals. In some cases the
metals do not reradiate all of the colors(energies)
and thus the metal has a color.Examples are gold
which retains some of the blue-green and thus appears
yellow and copper which retains some of the green and
appears reddish.

Iodine absorbs all colors and is thus black.
The combination of having both a smooth crystalline
surface and being black gives iodine it's shiny
appearance.

cyberbatt rated this answer Excellent or Above Average Answer

Question/Answer
itz_jacki asked on 02/26/03 - Stability of bonds in exothermic reaciton

which is more stable in an exothermic reaction? reactant bonds or product bonds? Does it have to do with the potential energy it possess?

matrix answered on 02/26/03:

When the PRODUCTS contain MORE STABLE bonds than the reactants ,the reaction is EXOTHERMIC.
When an energy-releasing chemical reaction occurs, bonds break and reactants atoms reorganize to form new bonds. The process yields products with different and more stable arrangements of their atoms. The products have less potential energy (chemical energy) than did the original reactants.The difference is the energy that is released in the forms of heat and light.

itz_jacki rated this answer Excellent or Above Average Answer

Question/Answer
tatiandarcia asked on 02/04/03 - Empirical formula

How do i calculate the theorical yield, in grams, of magnesium chloride?

matrix answered on 02/04/03:

Hello
You did not give me enough information.

To calculate theoretical yield you must give the reaction in which magnesium chloride is the product and also the amount of the starting materials.

Question/Answer
Navspert asked on 01/18/03 - Gas Pressure

Hello,

I would like to know ALL of the units (for example mmHg) for gas and atmospheric pressures. Please help as fast as you can, Thank you very much!

matrix answered on 01/19/03:

Hello
The common units used in chemistry are ;
atmosphere (atm),millimeters mercury (mm Hg),
inches Hg(in Hg),torr (same as mm Hg),kilopascals (kPa) and
pounds per square inch (psi).
Another unit you will see is bar (or millibar).

Here are some conversion factors for these units.
1 atm = 101,325 Pascals = 760 mm Hg = 760 torr = 14.7 psi.
101.325 kPa = 1 atm = 760 torr and 100 kPa = 1 bar = 750 torr
One millibar = 0.001 bar = 0.750 torr = 100 Pa
One bar = 750.062 torr = 0.9869 atm = 100,000 Pa.
Inches Hg(in Hg). Atmospheric pressure is 29.92 inches Hg


Normal atmospheric pressure is 14.7 psi.A column
of air one square inch in area rising from the
Earth's surface to space weighs 14.7 pounds.

torr - this unit is named after the scientist Torricelli,the inventor
of the barometer

Navspert rated this answer Excellent or Above Average Answer

Question/Answer
Navspert asked on 01/14/03 - TNT

Please help me with this ex. credit problem:

Write an unbalanced equation for the rapid combustion of Trinitrotoluene!

I can balance it myself.

Thank you very much!

matrix answered on 01/15/03:

Hello
I will give equations for both burning TNT and exploding TNT.

The unbalanced equation for the rapid COMBUSTION of
trinitrotoluene is below;

C7H5(NO2)3 + O2 ---> CO2 + H2O + N2



The unbalanced equation for the rapid DECOMPOSITION (explosion) of
trinitrotoluene is below;

C7H5O6N3(s) --> N2(g) + H2O(g) + CO(g) + C(s)



PLEASE RATE ALL ANSWERS YOU RECEIVE TO ALL OF YOUR QUESTIONS !

Navspert rated this answer Excellent or Above Average Answer

Question/Answer
Squall asked on 01/14/03 - radioactivity

What is the smallest element that is generally reguarded as being radioactive and whats its atomic weight?

matrix answered on 01/14/03:

Hello

The element with the lowest atomic mass that
is radioactive is tritium (atomic mass 3).
It is one of the the three isotopes of hydrogen.
The other two are protium [hydrogen] (atomic mass 1)
and deuterium (atomic mass 2).

Tritium is radioactive with a half-life of
12.5 years. It is used in nuclear reactors,
luminous paints and as a radiotracer.

Squall rated this answer Excellent or Above Average Answer

Question/Answer
Navspert asked on 01/02/03 - Bonds and stuff!

Hello, I yet some few more questions!

Which type of bond is stronger: Ionic or Covalent?

and

Are ionic groups like NaCl, KCl, LiCl, etc also called molecules? If not, what are they called? (I'm confused because I heard different)!

I really appreciate this, as you know, I'm only a beginner in chemistry!

Thank you.

matrix answered on 01/03/03:

Hello

Because these two types of bonds are so different, it is hard to make comparisons. Covalent bonds are considered to be SLIGHTLY STRONGER than ionic bonds.

http://www.launc.tased.edu.au/online/sciences/PhysSci/pschem/bondtype/BondTyp1.htm





Ionic compounds such as NaCl and KCl do NOT contain discrete molecules but instead consist of precise arrangements of alternating positive and negative ions. The number of positive and negative charges are equal so that the material has a neutral charge.

http://www.chem.uidaho.edu/~honors/ionbond.html

Navspert rated this answer Excellent or Above Average Answer

Question/Answer
Navspert asked on 12/31/02 - Balanced equation?

Hello, is the following chemical equation balanced for the complete combustion of octane? If not, please correct it :

C8H18 + (25/2)O2 ---> 8CO2 + 9H20

matrix answered on 01/01/03:

Hello Navspert

While numerically you have the correct number of
atoms on each side of the equation, the equation is
not balanced correctly. Correctly balanced equations do NOT use
fractions as coefficients.

The equation should be written as below.
2 C8H18 + 25 O2 ---> 16 CO2 + 18 H2O

http://learn.chem.vt.edu/tutorials/stoichiometry/balanceeq.html

Navspert rated this answer Excellent or Above Average Answer

Question/Answer
tatiandarcia asked on 12/17/02 - molecular forces

The Lewis structure of methane, CH4, contain how many covalent bonds and why?

matrix answered on 12/17/02:

Hello Tatiandarcia

Methane has 4 covalent bonds.

Carbon is the central atom in methane.It has 4 valence
electrons (electrons that can be used in bonding).Hydrogen
has 1 valence electron.Each of carbon's 4 valence electrons
will form a bonding pair with the hydrogen electrons.The central
carbon atom will therefore have no lone pairs of electrons and 4
bonding pairs of electrons.

Question/Answer
tatiandarcia asked on 12/17/02 - molecular forces

In a polar covalent bond, the electronegativity of two atom is zero?

matrix answered on 12/17/02:

Hello Tatiandarcia

I believe your question should be "In a polar covalent bond,
the DIFFERENCE in electronegativities of two atom is zero?"
The answer to this is that it is false.

The electronegativities of elements are not zero (see web
site below). When discussing the formation of bonds it is the
difference in electronegatities of the two atoms that is considered.

Periodic Table with Electronegativities
http://www.thecatalyst.org/electabl.html

Question/Answer
tatiandarcia asked on 12/17/02 - Covalent bonds

To draw a Lewis structure, it is not neccesary to know what?
A nonpolar covalent bond is unlikely to form between two atoms that differ in polarity? Is that true?

matrix answered on 12/17/02:

Hello tatiandarcia

It appears that you are giving me fragments of questions.
For example in your first question there are many things that
it is not neccesary to know.You need to give a list of possible
things in order to pick the one that is not neccesary.

QUESTION 1
This is how the first question is probably written.
"In order to draw a Lewis structure it is not necessary to know
a. bond energies
b. the types of atoms in the molecule
c. the number of valence electrons for each atom
d. the number of atoms in the molecule."
The answer is (a) bond energies

QUESTION 2
"A nonpolar covalent bonds is unlikely when two different atoms
join because the atoms are likely to differ in
a.electronegativity
b. density
c. state of matter
d. polarity"
The correct answer is (a)electonegativity

The question as you wrote it would be false.

Question/Answer
itz_jacki asked on 12/14/02 - Lead-acid battery

I just wander what are the anode, cathode and electrolyte in my lead-acid car battery. Do you know what it is?

matrix answered on 12/13/02:

Hello
The anode is made of porous lead metal.
The cathode is made of compressed lead oxide (PbO2).
The electrolyte is aqueous sulfuric acid.

Anode:
Pb(s) + SO4--(aq) ---> PbSO4(s) + 2e-

Cathode:
PbO2(s) + SO4--(aq) + 4 H+(aq) + 2e- ---> PbSO4(s) + 2 H2O(l)

Overall redox reaction:
PbO2(s) + Pb(s) + 4 H+(aq) + 2 SO4--(aq) ---> 2 PbSO4(s) + 2 H2O(l)


The battery is recharged when an electrical current is applied in the
opposite direction (generator or alternator), the electrodes are regenerated.
2 PbSO4(s) + 2 H2O(l) ---> Pb(s) + PbO2(s) + 4 H+(aq) + 2 SO4--(aq)

http://genchem.chem.wisc.edu/netorial/ROttosen/tutorial/modules/electrochemistry/06battery/18_63.html


itz_jacki rated this answer Excellent or Above Average Answer

Question/Answer
tatiandarcia asked on 12/03/02 - Chemistry

HI! . OK, HERE IS THE QUESTION:
SOME CRACKER TINS INCLUDE A GLASS VIAL OF DRYING MATERIAL IN THE LID. THIS IS OFTEN A MIXTURE OF MAGNESIUM SULFATE AND COBALT CHLORIDE. AS THE MIXTURE ABSOBS MOISTURE TO FORM HYDRATED COMPOUNDS, THE COBALT CHLOPINK CoCL2.6H2O. WHEN THIS HYDRATED MIXTURE BECOMES TOTALLY PINK, IT CAN BE RESTORED TO THE DIHYDRATE FORM BY BEING HEATED IN THE OVEN.
CAN YOU EXPLAIN TO ME HOW CAN I WRITE AN EQUATION FOR THE REACTIONS THAT OCCUR WHEN THIS MIXTURE IS HEATED PLEASE.!
i'D APPRECIATE YOUR ANSWER.

tHANKS.

matrix answered on 12/03/02:



Hello



Please use upper and lower case letters correctly! It is
very hard to read your questions when you use all upper
case letters. This is especially true in questions with
chemical synbols.






CoCl2�2H2O + 4 H2O ---> CoCl2�6H2O
CoCl2�6H2O (+ heat) ---> CoCl2�2H2O + 4 H2O



MgSO4 + 7 H2O ---> MgSO4�7H2O
MgSO4�7H2O (+ heat) ---> MgSO4 + 7 H2O
The MgSO4�7H2O needs to be heated to 250�C to remove ALL
the water .

Question/Answer
tatiandarcia asked on 12/01/02 - Traditional and stock names

Hi!, I'd like somebody explain to me how can i get the traditional names and stock names for the following formulas:
Hg2S, HgCO3, Hg3(PO4)2, (Hg2)3(PO4)2

matrix answered on 12/01/02:


Hello
Hg2S...mercurous sulfide...mercury(I) sulfide
HgCO3...mercuric carbonate...mercury(II) carbonate
Hg3(PO4)2...mercuric phosphate...mercury(II) phosphate
(Hg2)3(PO4)2...mercurous phosphate...mercury(I) phosphate



Traditional names.
mercury in +1 oxidation state is mercurous.
mercury in +2 oxidation state is mercuric.



Stock names.
the oxidation state is indicated by Roman numberals in parenthesis.



http://dbhs.wvusd.k12.ca.us/Nomenclature/Binary-Stock-FormulatoName.html

Question/Answer
itz_jacki asked on 11/30/02 - Galvanic and electrolytic cells

Are there half cells for electrolytic cells?

matrix answered on 12/01/02:


Hello

Electrolytic cells are similar to galvanic cells as they
are composed of two half-cells,one a reduction half-cell and
the other an oxidation half-cell. The difference between a galvanic
cell and an electrolytic cell is that the direction of both half-reactions
have been reversed. It is reversed because an external power source with
potential greater than then that of the galvanic cell has been placed on
the cell .

Volatic cell
Zn(s) + Cu+2(aq) -> Zn+2(aq) + Cu(s) E0 = 1.101 V
Zn(s) ---> Zn++(aq)
Cu++(aq) ----> Cu(s)

Electrolytic cell
Zn+2(aq) + Cu(s) -> Zn(s) + Cu+2(aq) E0 = -1.101 V
Zn+2(aq) ---> Zn(s)
Cu(s) ---> Cu+2(aq)

http://genchem.chem.wisc.edu/netorial/ROttosen/tutorial/modules/electrochemistry/07electrolysis/18_71.html
http://chemed.chem.purdue.edu/genchem/topicreview/bp/ch20/faraday.html




itz_jacki rated this answer Excellent or Above Average Answer

Question/Answer
tatiandarcia asked on 11/23/02 - IONIC COMPOUND

FOR THE FOLLOWING WORD LIST, I'D LIKE YOU HELP ME WITH THESE QUESTIONS; PLEASE: BINARY/ TERNARY DIFFERENT/ EQUAL ELECTRONS/ PROTONS EMPIRICAL/ MOLECULAR GAIN/ LOSE IONS/ MOLECULES ONE/ TWO POSITIVE/ NEGATIVE SOLID/ LIQUID SULFIDE ALUMINIUM SODIUM MAGNESIUM POTASSIUM AN ION IS AN ATOM OR GROUP PF ATOMS THAT HAS ACQUIRED A CHARGE. tHE CHARGE IS DUE TO THE PRESENCE OF________NUMBERS OF __________AND ELECTRONS IN EACH ATOM. ATOMS OF METALS USUALLY___ELECTRONS TO BECOME IONS WITH_____CHARGE. ATOMS OF NONMETALS USUALLY___________ELECTRONS TO BECOME IONS WITH______CHARGE. SOME NATURALLY OCCURING COMPOUNDS ARE IONIC COMPOUNDS. THAT IS, THEY CONSIST OF POSITIVE AND NEGATIVE__________ RATHER THAN ATOMS AND/ OR MOLECULES. IONIC COMPOUNDS ARE ALWAYS_________AT ORDINARY TEMPERATURES. THE POSITIVE ABD NEGATIVE___________THAT MAKE UP THE COMPOUND ARE LOCATED AR FIXED POINT WITHIN THE CRYSTALLINE SOLID. THESE_______DO NOT FORM MOLECULES. BECAUSE NO MOLECULES ARE PRESENT, CHEMICAL FORMULAS FOR IONIC COMPOUNDS MUST BE _____FORMULAS. sOME EXAMPLES OF SUCH IONIC COMPOUNDS WITH THEIR EMPIRICAL FORMULAS ARE__________OXIDE, Na2O, _____bROMIDE, MgBr2, AND _______CHLORIDE, alCL3. tHE EMPIRICAL FORMULA FOR _______SULFIDE IS K@S. THIS MEANS THAT THE IONIC SOLID, ____________________ SULFDE, CONSISTS OF POTASSIUM IONS AND _________ IONS LOCATED AT FIXED POINTS WITHIN THE CRYSTAL IN A RATIO OF_________________ TO ONE. SUCH COMPOUNDS, WHICH HAVE TWO KINDS OF ELEMENTS, ARE CALLED_____________COMPOUNDS. plEASE, PLEASE HELP ME WITH THIS. I'D APPRECIATE IT.! THANKS.:>)

matrix answered on 11/24/02:

Please use upper and lower case letters correctly! It is very hard to read your questions when you

use all upper case letters.

For the following word list, I'd like you help me with these questions please.

Binary

ternary

different

equal

electrons

protons

empirical

molecular

gain

lose

ions

molecules

one

two

positive

negative

solid

liquid

sulfide

aluminium

sodium

magnesium

potassium

An ion is an atom or group of atoms that has acquired a charge. The charge is due to the

presence of different numbers of protons and electrons in each atom.

Atoms of metals usually lose electrons to become ions with positive charge.

Atoms of nonmetals usually gain electrons to become ions with negative charge.

Some naturally occuring compounds are ionic compounds. That is, they consist of positive and negative ions

rather than atoms and/ or molecules.

Ionic compounds are always solid at ordinary temperatures.

The positive and negative ions that make up the compound are located at fixed points within the crystalline solid.

These "compounds" do not form molecules. Because no molecules are present, chemical formulas for ionic compounds must be

empirical formulas.

Some examples of such ionic compounds with their empirical formulas are sodium oxide, Na2O, magnesium bromide, MgBr2, and

aluminum chloride, AlCl3.

The empirical formula for potassium sulfide is K2S.

This means that the ionic solid, potassium sulfide, consists of potassium ions and sulfide ions located at fixed points within the

crystal in a ratio of two to one.

Such compounds, which have two kinds of elements, are called binary compounds.

tatiandarcia rated this answer Excellent or Above Average Answer

Question/Answer
_JacquelineA asked on 11/20/02 - Pigmentation of Skin

I read a true story of this man who was white and had changed the pigmentation of his skin to black during the 1950's to do an observation in the south so he could make history of what he's done. I would like to do the same so I can make an observation for myself in the present day and I'd like to then write a book on it. What I want to know is where can I find more information on doing this, what will the costs be, and exactly how do I do this. Thanks.

matrix answered on 11/20/02:

Hello John Howard Griffin described in his book BLACK LIKE ME about changing his pigmentation to pass as a black.He used chemicals called psoralens. They work by making the skin hypersensitive to the sun. To counteract skin damage, melanin production is accelerated. Griffin took psoralens in conjunction with medical grade UV lamp exposure to darken his skin.Psoralens do not make the skin darker without exposure to UV, and that exposure must be carefully regulated. This procedure is called the PUVA treatment. You will need to do this under a doctor's supervision and that will determine the costs. The hazards of the treatment have caused many dermatologists to recommend it only for patients with dark skin, never for people with red or blond hair, blue eyes, etc. The psoralens used for this are trimethylpsoralen, 8-methoxypsoralen and methoxsalen. The structures of some of these compounds are shown on the following site. http://www.geocities.com/a_chernyshev/vitiligo.html

_JacquelineA rated this answer Excellent or Above Average Answer
s1n1st3r_ur9e rated this answer Excellent or Above Average Answer

Question/Answer
itz_jacki asked on 11/12/02 - Reduction and Oxidation

Can you explain to me in a equation, for example H2 + Cl2 --> 2HCl, which is the reducing agent, which is the oxidizing agent, which is being reduced, and which is being oxidized. I am very confused.... thx.

matrix answered on 11/12/02:

Hello When an element is oxidized ,it's oxidation number increses. When an element is reduced ,it's oxidation number decreases. A substance that reacts with a second substance and increases the oxidation of the SECOND substance is an oxidizing agent.In this reaction the oxidizing agent is reduced. YOU CANNOT HAVE OXIDATION OF A ELEMENT WITHOUT ALSO HAVING REDUCTION OF A SECOND ELEMENT. Something has to give electrons and something else MUST be receiving these electrons ! H2 + Cl2 ---> 2 HCl In your reaction the hydrogen is going from oxidation number 0 to +1. It is being oxidized.The chlorine is the oxidizing agent and it is being reduced.The chlorine is going from 0 to -1. The hydrogen is the reducing agent. Please read the following sites for more help. http://www.howe.k12.ok.us/~jimaskew/credox.htm http://cator.hsc.edu/~kmd/caveman/projects/rxn.2/ http://home.att.net/~lfretzin/notes14.htm

itz_jacki rated this answer Excellent or Above Average Answer

Question/Answer
slimorish2003 asked on 10/28/02 - oxygen

02 + 4e- ---> 20^2- Usually I only see 2 electrons being donated why does oxygen donates 4 electrons?

matrix answered on 11/06/02:

Hello 02 + 4 e- ---> 2 0^2- A molecule of O2 contains two atoms of oxygen. Each atom is going from 0 to -2. Thus a total of 4 electrons are involved in this reaction.

slimorish2003 rated this answer Excellent or Above Average Answer

Question/Answer
sunkissed02 asked on 11/05/02 - Balancing

How do you balancing this question? I've been spending hrs trying to balance this equation. C12H12O11 + H20---> C2H5OH+ CO2

matrix answered on 11/06/02:

Hello C12H22O11(aq) + H2O(l) ��>4 C2H5OH(aq) + 4 CO2(g) This reaction goes when you have yeast present.This is the equation for the fermentation of sucrose.

sunkissed02 rated this answer Excellent or Above Average Answer

exper   © Copyright 2002-2008 Answerway.org. All rights reserved. User Guidelines. Expert Guidelines.
Privacy Policy. Terms of Use.   Make Us Your Homepage
. Bookmark Answerway.